Posts Tagged ‘Bible archaeology’

h1

The Date of the Exodus (2/2)

February 19, 2015

This article is the second of two in a consideration of the date of the Exodus. Typically, only two dates are considered viable; c.1440 BCE (the ‘early date’), and c.1280 BCE (the ‘late date’). Arguments for these dates are reviewed and compared here.

Summary of Key Arguments

Here are the key arguments for the early date (c. 1440 BCE), together with criticisms.

Argument: The 480 years of 1 Kings 1:6 indicates an early date

  • This is only a relative chronology, and the number does not agree with the years recorded in the Judges[1] [2]
  • The number may be symbolic for 12 generations[3]
  • This disagrees with the date for Abraham[4]

Argument: Some destruction layers in Canaan support an early date[5]

  • Destruction and occupation layers provide more support for a late date[6]
  • No evidence for Edom and Moab existing at an early date[7]

Argument: Reference to the Habiru in the 14th century Armana letters[8]

  • Extensive study has revealed no direct correspondence between the Habiru and the Hebrews, thus the Armana letters do not support the early date[9] [10]
  • Armarna correspondence contradicts early date destruction of Hazor[11]

Argument: No evidence for 13th century occupation or destruction of Jericho, Ai, or Hazor[12]

  • Hazor was occupied and destroyed in the 13th century,[13] reliable excavation of 13th century Jericho is challenged by extensive erosion,[14] and the archaeological data for Ai is difficult to reconcile with both early and late dates;[15] these are insignificant archaeological challenges for the late date

Here are the key arguments for the late date (c. 1280 BCE), together with criticisms.

Argument: Destruction of Hazor in 13th century[16]

  • The archaeological evidence may indicate the destruction in Judges 4:24[17]

Argument: Pithom and Rameses in Exodus 1:11 are evidence for events under Pharaoh Rameses II (1279-1213 BCE)[18] [19]

  • This requires Rameses to be built before Rameses II even began to rule[20]
  • Unlike the pharaoh of the Exodus, Rameses II did not die in the Red Sea[21]
  • The sites were built earlier; one was later renamed ‘Rameses’, and an editor of the book of Exodus updated the text with this name[22]

Argument: Covenant formulas in the Law of Moses closely match those from 1400-1200 BCE[23]

  • There is an insufficient match to date the Biblical covenants precisely[24]

Argument: The Merneptah Stele (13th century), is the earliest reference to Israel in Canaan[25]

  • Egyptologist Manfred Görg has suggested an Egyptian inscription he dates to the 13th century, contains a reference to Israel which may have been copied from an 18th Dynasty record (16th-13th centuries BCE), implying Israel was in Canaan before the 13th century[26]

Argument: Egypt occupied Canaan until the 12th century[27]

  • This remains unaddressed by key proponents of the early date;[28] this contradicts completely a late date for the Exodus

Argument: A very large population entered Canaan in the late 13th century[29]

  • This remains unaddressed by key proponents of the early date; [30] there is no evidence for such a population entering Canaan in the 15th century

Review of Arguments

The early date is highly vulnerable to a range of criticisms, and has the least archaeological support. In particular, the occupation and control of Canaan by Egypt until the end of the 13th century, the lack of any evidence for a new population entering Canaan in the 15th century, the Armana correspondence, the non-existence of Edom and Moab in the 15th century, and the evidence for destruction of Canaanite sites matching a 13th century conquest rather than a 15th century conquest, are  formidable challenges to the traditional late date.

Objects to the late date are less substantial. There is no evidence that Rameses as a place name in Exodus 1 is a later editorial gloss.[31] Görg’s suggestion of an Egyptian reference to Israel earlier than the Merneptah Stele is problematic. [32] [33] Wood’s attribution of the 13th century Hazor destruction level to Deborah and Barak fails to provide evidence.[34] [35] His dating of Pithom and Rameses on the basis of the birth of Moses being described later in Exodus 1, assumes an unnecessarily strict chronological sequence for the narrative.

The pharaoh under whom Pithom and Rameses were built died while Moses was in the wilderness before the Exodus[36] (matching Rameses II and a late date Exodus). Additionally, Hoffmeier argues Exodus does not represent the pharaoh of the Exodus as dying in the Red Sea,[37] whereas an early date pharaoh would have to be Thutmose III or Amenhotep II, neither of whom died by drowning.[38]

Conclusion

Although vigorous debate over date of the Exodus is ongoing, the 13th century date continues to be held widely among those scholars who accept the historicity of the Exodus.[39] As early as 1999 Hoffmeier observed ‘Dating the period of the oppression and exodus to the fifteenth century B.C. has largely been replaced in favor of a thirteenth-century date’.[40]


[1] The years of the judges, if added sequentially, result in 633-650 years between the exodus and the reign of Solomon, compelling supporters of the early date to read the years in Judges less literally, in order to read the years in 1 Kings 6:1 more literally; ‘To get around the dilemma caused by the difference between 480 and 633-650 years, advocates of the 15th-century (and the later date) exodus date are forced to harmonize the conflicting data by proposing some overlap between judgeships to bring the 480-year figure into alignment with the 633–650 year total.12  By doing this, one abandons a straightforward, literal reading of the Judges through Exodus narratives.’, Hoffmeier, ‘What is the Biblical Date For the Exodus? A Response to Bryant Wood’, Journal of the Evangelical Theological Society, 50/2 (2007), 228.

[2] ‘When one seeks to reconstruct the numbers given in the biblical accounts, consistently and literally, they do not add up to the number 480 given in 1 Kgs 6:1.’, Hawkins, ‘Propositions For Evangelical Acceptance Of A Late-Date Exodus-Conquest:  Biblical Data And The Royal Scarabs From Mt. Ebal’, Journal of the Evangelical Theological Society, 50/1 (2007), 35.

[3] ‘It has long been thought that the 480-year figure of 1 Kgs 6:1 might be a symbolic figure that derives from 12 times 40-40 years being a symbolic number for a generation—thus signifying that 12 generations had elapsed between the exodus and Solomon’s 4th year. Since men were usually married and had children by age 20–25, 60  a period closer to 300 years would be more accurate. When one adds 300 to 967 BC, an Exodus date around 1267 BC (20 years into the reign of Ramesses II) results.’, ibid., p. 236.

[4] ‘A 15th-century B.C. date presents problems for the chronology of Abram. Archaeological evidence relating to the overthrow of Sodom and Gomorrah seems to date Abram’s arrival in Canaan around 1900 B.C. The Genesis narratives place Jacob’s migration to Egypt about 215 years later. On the basis of the 430 years of Exodus 12:40 it would seem that Abram came to Canaan about 2086 B.C., some 645 years before the exodus. That would date his birth (cf. 12:4) about 2161 B.C. If the Sodom and Gomorrah evidence is correct, Abram’s arrival in Canaan would harmonize with a 13th-century B.C. date.’, Harrison, ‘Exodus, The’, in Elwell & Beitzel, Baker Encyclopedia of the Bible (1988), 743-744.

[5]According to Wood, some archaeological findings—such as destruction layers from Jericho, Ai and Hazor—support a 15th-century exodus (Wood, “The Rise and Fall of the 13th-Century Exodus-Conquest Theory,” 488–89; Wood, “From Ramesses to Shiloh,” 256–82).’, Thornhill, ‘Exodus’, in Barry & Wentz (eds.), The Lexham Bible Dictionary (2012).

[6] ‘The destruction and occupation layers of many conquest cities (e.g., Lachish, Debir, Hazor, Bethel, etc.) favor the 13th-century dating.’, ibid.

[7] ‘Excavation findings seem to indicate that Edom and Moab (compare Exod 13:15; Num 20:14–21) were not yet established peoples during the mid-14th century.’, ibid.

[8] ‘Wood also cites the mention of the ‘ (‘)apiru in the Canaanite Amarna letters of the mid-14th century, as well as an inscription dating to the 18th Dynasty. This inscription appears to mention Ashkelon, Canaan, and Israel (Wood, “The Rise and Fall of the 13th-Century Exodus-Conquest Theory,” 489).’, Thornhill, ‘Exodus’, in Barry & Wentz (eds.), The Lexham Bible Dictionary (2012).

[9] ‘The relationship between the “Habiru” of the Amarna letters, the “Apiru” in 13th-century B.C. Egypt, and the biblical Hebrews has been examined minutely by scholars. Widely differing opinions have been offered. Some believe that the three are variations of the name of one people. To others, however, it seems far from clear that there was any significant relationship between the names. Such disagreement also tends to intensify the problem.’, Harrison, ‘Exodus, The’, in Elwell & Beitzel, Baker Encyclopedia of the Bible (1988), 744.

[10] ‘The ʿapiru (sometimes ḫapiru or ḫabiru) are considered to be warlords, brigands and disenfranchised peoples on the outskirts of society. Rainey has demonstrated that the term cannot be etymologically related to “Hebrew,” and the range of use of the term makes it clear that the ʿapiru cannot be equated with Israelites. Nevertheless, some would contend that it does not entirely rule out the possibility that Israelites, along with other peoples, could have been designated by the term.’, Walton, ‘Exodus, Date of’, in Alexander & Baker, Dictionary of the Old Testament: Pentateuch (2003), 263.

[11]According to the Wood, the marauding Habiru of the Amarna Letters could be the Hebrews.113  Abi-Milku, however, makes clear that Hazor was an ally of the Habiru rather than being the destroyers of Hazor. This information from the Amarna correspondences demonstrates that Hazor during the LB IIA was a major player in the region and does not sound like a city that had just been demolished and burnt by Joshua and his forces.’, Hoffmeier, ‘What is the Biblical Date For the Exodus? A Response to Bryant Wood’, Journal of the Evangelical Theological Society, 50/2 (2007), 245.

[12] ‘Only three cities are recorded as having been destroyed by fire by the Israelites: Jericho (Josh 6:24); Ai (Josh 8:28); and Hazor (Josh 11:11).14  All three pose problems for a late 13th-century conquest. At Jericho and Ai, no evidence has been found for occupation in the late 13th century, let alone for a destruction at that time.’, Wood, ‘The Rise and Fall of the 13th-Century Exodus-Conquest Theory’, Journal of the Evangelical Theological Society, 48/3 (2005), 477.

[13] ‘In Canaan, the drastic destruction of Hazor (level 13) in the later 13th century B.C. (despite misconceptions to the contrary) may well reflect Joshua’s exploit.’,  Kitchen, ‘Exodus, The,’, in Freedman (ed.), Anchor Yale Bible Dictionary (1992), 702.

[14] ‘at Jericho, nearly half a millennium of erosion has long since removed virtually all pertinent evidence.’, ibid., p. 702.

[15] ‘Ai remains an enigma on any view;’, ibid., p. 702.

[16] ‘In Canaan, the drastic destruction of Hazor (level 13) in the later 13th century B.C. (despite misconceptions to the contrary) may well reflect Joshua’s exploit.’,  Kitchen, ‘Exodus, The,’, in Freedman (ed.), Anchor Yale Bible Dictionary (1992), 702.

[17]Following the 1230 bc destruction, there was no urban center there until the time of Solomon in the 10th century bc (1 Kgs 9:15).16  The defeat of Jabin, king of Hazor, by a coalition of Hebrew tribes under the leadership of Deborah and Barak is recorded in Judges 4–5. Judges 4:24 indicates that the Israelites destroyed Hazor at this time: “And the hand of the Israelites grew stronger and stronger against Jabin, the Canaanite king, until they destroyed him.”17  If Joshua destroyed Hazor in 1230 bc, then there would be no city for the Jabin of Judges 4 to rule.’, Wood, ‘The Rise and Fall of the 13th-Century Exodus-Conquest Theory’, Journal of the Evangelical Theological Society, 48/3 (2005), 477.

[18] ‘Egyptologists have long understood the reference to Rameses to refer to Pi-Ramesses, the delta metropolis built by Ramesses II, the 19th Dynasty monarch who reigned from 1279–1213 BC.’, Hoffmeier, ‘What is the Biblical Date For the Exodus? A Response to Bryant Wood’, Journal of the Evangelical Theological Society, 50/2 (2007), 231.

[19] ‘The archaeological data is now unequivocal: Pi-Ramesses is located at modern-day Qantir, near Faqus, and was built by Ramesses II beginning around 1270 BC;’, ibid., pp. 232-233.

[20] ‘Since Moses was 80 years of age at the time of the exodus (Exod 7:7), the building of Rameses would have taken place well before Moses’ birth in 1340 bc (according to the 13th-century theory), long before Rameses came to the throne.’, Wood, ‘The Rise and Fall of the 13th-Century Exodus-Conquest Theory’, Journal of the Evangelical Theological Society, 48/3 (2005), 478.

[21] ‘Obviously, Rameses II did not drown in the yam sup, [commonly translated ‘Red Sea’] as he died of natural causes some 47 years after the presumed exodus date of 1260 bc.’, ibid., p. 478.

[22] ‘It is clear, then, that the name Rameses used in Exod 1:11 is an editorial updating of an earlier name that went out of use.’, ibid., p. 478.

[23] ‘Scholars have understood for some time, since the work of Mendenhall, Kline, and Kitchen, that the book of Deuteronomy has the literary and legal form that characterized late second millennium BC Hittite international treaties.’,  Niehaus, ‘Covenant and Narrative, God and Time’, Journal of the Evangelical Theological Society 53/3 (2010), 550.

[24] ‘The format of the biblical material is varied and complex and cannot be dated to a particular time period based on ANE treaty documents’, Wood, ‘The Rise and Fall of the 13th-Century Exodus-Conquest Theory’, Journal of the Evangelical Theological Society, 48/3 (2005), 480-481.

[25] ‘The Merneptah stela is also cited as evidence for this date, since Israel is referenced as a people group rather than a nation.’, Thornhill, ‘Exodus’, in Barry & Wentz (eds.), The Lexham Bible Dictionary (2012).

[26] ‘Due to the similarity of these names to the names on the Merenptah stela, Gorg suggests the name list may derive from the time of Rameses II, but adopting an older name sequence from the 18th Dynasty. This evidence, if it holds up to further scrutiny, would also support a 15th-century bc exodus-conquest rather than a 13th-century bc timeframe.’, Wood, ‘The Rise and Fall of the 13th-Century Exodus-Conquest Theory’, Journal of the Evangelical Theological Society, 48/3 (2005), 489.

[27] ‘In trying to work out an evangelical understanding of the emergence of Israel, Mark Chavalas and Murray Adamthwaite have recently noted that certain conditions in the archaeology of Palestine appear to mitigate against the traditional early date positioning of the Exodus/Conquest.12  They note that, at a series of sites all over Palestine, “the clear picture is that Egyptian occupation continued until the end of the Late Bronze Age (1200 BC).”’, Hawkins, ‘Propositions For Evangelical Acceptance Of A Late-Date Exodus-Conquest:  Biblical Data And The Royal Scarabs From Mt. Ebal’, Journal of the Evangelical Theological Society, 50/1 (2007), 34.

[28] For example, it is never mentioned by Bryant Wood (foremost proponent of the early date), in his key articles ‘The Rise and Fall of the 13th-Century Exodus-Conquest Theory’, Journal of the Evangelical Theological Society, 48/3 (2005), 475-489, and ‘The Biblical Date For The Exodus Is 1446 BC:  A Response To James Hoffmeier’, Journal of the Evangelical Theological Society, 50/2 (2005), 249-258.

[29] ‘The implication seemed clear that a new population group had arrived in the Central Hill-Country during the transition from the Late Bronze Age to the Iron Age I.’, Hawkins, ‘Propositions For Evangelical Acceptance Of A Late-Date Exodus-Conquest:  Biblical Data And The Royal Scarabs From Mt. Ebal’, Journal of the Evangelical Theological Society, 50/1 (2007), 34.

[30] ‘While this material has seemed to point toward a late date for Israel’s emergence in Canaan,9  it has largely gone unnoticed by evangelical scholars writing histories of Israel10  or commentaries on Joshua.’, ibid., p. 34.

[31] ‘The toponym Rameses (רַעַמְס) occurs five times in the OT, in Gen 47:11; 53  Exod 1:11; 12:37; and Num 33:3, 5. In none of these cases is the formula “old name +הוא+ new name” used, nor does a longer explanatory gloss with the word לָרִאשֹׁנה—“at the first” occur with any of the five citations. In other words, there is no evidence within these five passages to suspect that “Rameses” is an editorial gloss.’, Hoffmeier, ‘What is the Biblical Date For the Exodus? A Response to Bryant Wood’, Journal of the Evangelical Theological Society, 50/2 (2007), 234.

[32] ‘Görg’s reading of this name as “Israel” is plagued by serious linguistic and orthographic problems that preclude it from being Israel.’, Hoffmeier, ‘What is the Biblical Date For the Exodus? A Response to Bryant Wood’, Journal of the Evangelical Theological Society, 50/2 (2007), 241.

[33] ‘Especially given the absence of Israel from the Armana evidence, this seems intrinsically unlikely, given the early date and lacking a full reading.’, Fleming, The Legacy of Israel in Judah’s Bible: History, Politics, and the Reinscribing of Tradition (2012), 241.

[34] ‘A close reading of the text indicates that God gave Israel victory over her oppressors in a major battle 25 miles away from Hazor, but the text is absolutely silent regarding any military action against Hazor itself. Furthermore, the terminology used in 4:23–24 is not found in Joshua or Judges to indicate attacks on cities. Consequently, there is no basis to believe that the destruction of the final LB IIB (late 13th century) city was caused by Deborah and Barak’s triumph over Jabin and Sisera’, Hoffmeier, ‘What is the Biblical Date For the Exodus? A Response to Bryant Wood’, Journal of the Evangelical Theological Society, 50/2 (2007), 244.

[35] ‘From the Amarna letters, written to the pharaohs Amernhotep III and Akhenaten between 1390–1340 BC, we learn that Hazor was thriving during this period.’, ibid., p. 245.

[36] Exodus 2:23 During that long period of time the king of Egypt died, and the Israelites groaned because of their slave labor. They cried out, and their desperate cry because of their slave labor went up to God.

[37] ‘Psalm 136:15 may be the closest to suggest that pharaoh drowned in the seas, but that may be due to misleading English translations, e.g. JB: “Drowned Pharaoh and his army”; NIV: “swept pharaoh and his army into the Red Sea”; KJV and NAS: “He overthrew Pharaoh. .. into the Red Sea.” The key word here is נאר, which is the word used in Exod 14:27. נאר, means to “shake off” (Ludwig Koehler and Walter Baumgartner, The Hebrew and Aramaic Lexicon of the Old Testament [Leiden: Brill, 2001] 707). Nothing in this term suggests that pharaoh drowned in the sea. In fact, there is nothing to suggest in the various texts, especially in Exodus, that pharaoh led the chariot corps in pursuit of the escaping Hebrews. Perhaps people have been influenced by Cecil B. DeMille’s portrait of angry Ramesses (Yul Brynner) leading the attack at the sea. But even in The Ten Commandments, Ramesses does not follow the Israelites into the sea!’, Hoffmeier, ‘What is the Biblical Date For the Exodus? A Response to Bryant Wood’, Journal of the Evangelical Theological Society, 50/2 (2007), 239.

[38] ‘The second problem for Wood’s exodus pharaoh drowning in the sea is that the mummy of Thutmose III was found in the Deir el-Bahri cache, while Amenhotep IIs was actually discovered in his tomb, one of only a few royal mummies discovered intact.81  In fact, all the mummies of the 15th century are accounted for.82  According to the X-rays and investigations of these mummies, none indicate a death by drowning.’, ibid., p. 240.

[39] ‘The need for discussing the latter premise is that many biblical scholars who affirm the historicity of the exodus now date it to the thirteenth century B.C., questioning concrete numbers in the Bible that taken literally would place the exodus in the fifteenth century B.C.’, Petrovich, ‘Amenhotep II And The Historicity Of The Exodus-Pharaoh’, The Master’s Seminary Journal, 17/1 (2006), 83.

[40] ‘Dating the period of the oppression and exodus to the fifteenth century B.C. has largely been replaced in favor of a thirteenth-century date, although a few adherents to the earlier date have followed Jack’s thesis.’, Hoffmeier, Israel In Egypt: The Evidence for the Authenticity of the Exodus Tradition (1999), 125.

h1

The Date of the Exodus (1/2)

February 19, 2015

Despite over a century of detailed investigation, the date of the Hebrew exodus from Egypt remains a topic of extensive debate within scholarship.[1] Scholarly discussion focuses on exegetical concerns such as the interpretation of chronological data in the Old Testament, the correct identification of toponyms (place names), and the relationship between textual and archaeological data. This initial article summarizes the emergence of the two most commonly proposed dates.

History of Interpretation

Throughout the 19th century, Rameses II was considered the pharaoh under whom the Hebrews were enslaved, and his son Merneptah the pharaoh of the exodus.[2]  However, discovery of the Merneptah Stele, referring to Israel as a recognized people settled in Canaan by the 14th century, invalidated this view.

‘This new data appeared to require that Israel had already been settled there by the end of the 13th century BC. Placing Israel in Canaan this early in the reign of Merneptah raised obstacles for his having been the pharaoh of the Exodus. Israel obviously could not have left Egypt in the first year of Merneptah’s reign, wandered in the wilderness for forty years, and then appeared in Canaan as a settled ethnic group in his fifth year.’[3]

A revised interpretation identified Ramases II as the pharaoh of the exodus, a view which remained dominant throughout the rest of the 19th century, up to the 1920s.[4] [5] In 1925 a 15th century date was proposed.

‘This approach seems to have been pioneered initially by James Jack, who challenged the 13th-century BC date in his 1925 book, The Date of the Exodus in the Light of External Evidence. Jack argued that both biblical and extrabiblical evidence pointed to a mid-15th century BC date.’[6]

Textual Evidence

The early date rests principally on an application of the chronology given in 1 Kings 6:1, which appears to date the exodus 480 years before the reign of Solomon.[7] Since there is considerable agreement that Solomon’s reign started at around 960-960 BCE, counting 480 years back from this date places the exodus at c.1440 BCE. A second text seen as corroborating the early date is Judges 11:26, in which Jephthah’s claim that Israel had already been in Canaan for at least three hundred years[8] would appear to suggest an early date for the exodus.[9]

The late date receives textual support from Exodus 1:11, which refers specifically to the Hebrews building Pithom and Rameses for the Pharaoh.[10] Unlike 1 Kings 6:1 and Judges 11:26, this text rests on an absolute rather than a relative date; the construction of buildings at Pithom and Rameses.

‘Late-date theorists argue that, since the Exodus account used the name by which the city was known for about two centuries only (c. 1300–1100 B.C.), the Hebrew tradition of the exodus must also date from that period. In such an event Rameses II would have been the pharaoh of the oppression, and his son Merneptah (1224?–1214 B.C.) the pharaoh of the exodus.’[11]

Archaeological Evidence for the Conquest

In the absence of direct archaeological evidence for the Hebrew settlement in Goshen, and the lack of Egyptian records describing the Hebrews as an enslaved ethnic group, or the plagues, or subsequent exodus, attempts to date the exodus using archaeological evidence focused on dating the Hebrew entry into Canaan, searching for evidence of conquest.

Attempts have been made by proponents for both dates, and interpretation of the archaeological record has been much contested. In the 1930s, archaeologist John Garstang’s excavations of Tell es-Sultan led him to conclude there was strong evidence for a Hebrew destruction of Jericho before 1400 BCE, lending weight to an early date exodus.[12]

However, Kathleen Kenyon’s subsequent investigation of the site re-dated the destruction to around 1500 BCE, too early for the Hebrews.[13] Efforts by Bryant Wood to defend Garstang’s dating and attribute the destruction of Jericho to an early date Hebrew conquest,[14] [15] have not gained any significant scholarly acceptance outside evangelical circles. Contemporary with Garstang, archaeologist Wiliam Albright arrived at a late date for the exodus, on the basis of his investigations of Canaanite archaeological sites.[16]

In response to acknowledged difficulties harmonizing an early date with the archaeological record, in the 1970s John Bimson proposed a Hebrew conquest during the end of the Middle Bronze Age, which seemed to fit the Biblical record more closely.[17] However, Bimson’s interpretation was critiqued strongly by many scholars.

‘The critique of Bimson’s proposal came from numerous quarters. Bietak objected that his suggested alteration was only fifty years, therefore still in the sixteenth century, and could not be stretched as far as Bimson needed it to be. B. Halpern objected that the changes suggested by Bimson would leave a reduced time span for LB I that could not possibly accommodate the archaeological data.’ [18]

Archaeological evidence for destruction and occupation layers supporting a late date exodus, was considered more abundant.

‘Archaeological evidence from Canaanite sites such as Bethel, Debir, Lachish, and Hazor indicates destruction at 13th-century B.C. levels, a fact generally regarded as relating to the Hebrew occupation under Joshua.’[19]

Evidence from Philistine sites tends to favor a late rather than an early date for the exodus. Against that, however, must be set the fact that the major Philistine occupation of the southern Palestinian coastlands only occurred around 1175 B.C., in the time of Rameses III.’[20]

The Ongoing Dispute

By the 1970s the date of the exodus had ceased to become a significant concern within critical scholarship, as many commentators no longer believed in the essential historicity of the event. However, the issue continued to be debated hotly among evangelical and other faith professing scholars, as well as among a minority report of critical scholars and those professional archaeologists who considered the Biblical exodus account to preserve an essentially accurate historical core.

The second article in this series will compare and contrast the evidence and arguments advanced for each date, together with their respective counter-arguments.


[1]The date of the Exodus is one of the most debated topics in OT studies because of the ambiguous nature of the evidence.’, Shea, ‘Exodus, Date of the’, in Bromiley et al. (eds.), The International Standard Bible Encyclopedia, Revised, volume 2 (1979–1988), 230.

[2] ‘With the identification of Ramesses II as the pharaoh of the oppression, his son Merneptah, who succeeded him on the throne, naturally became the pharaoh of the Exodus. Based on this reasoning, the biblical Exodus was securely located by scholars within the 19th Dynasty of Egypt (1293–1185 BC) throughout the nineteenth century.’, Hawkins, ‘Propositions For Evangelical Acceptance Of A Late-Date Exodus-Conquest:  Biblical Data And The Royal Scarabs From Mt. Ebal’, Journal of the Evangelical Theological Society, 50/1 (2007), 31-32.

[3] Ibid., p. 32.

[4] ‘Up until about 1925, this position was widely held by scholars, both evangelical and otherwise.’, ibid., p. 32.

[5]At the beginning of the 20th century many scholars, both liberal and conservative, placed the date toward the end of the 13th century B.C.’, Harrison, ‘Exodus, The’, in Elwell & Beitzel, Baker Encyclopedia of the Bible (1988), 742.

[6] Hawkins, ‘Propositions For Evangelical Acceptance Of A Late-Date Exodus-Conquest:  Biblical Data And The Royal Scarabs From Mt. Ebal’, Journal of the Evangelical Theological Society, 50/1 (2007), 33.

[7] 1 Kings 6:1 In the four hundred and eightieth year after the Israelites left Egypt, in the fourth year of Solomon’s reign over Israel, during the month Ziv (the second month), he began building the LORD’s temple.

[8] Judges 11:26 Israel has been living in Heshbon and its nearby towns, in Aroer and its nearby towns, and in all the cities along the Arnon for three hundred years! Why did you not reclaim them during that time?

[9] ‘If 1100 BC is taken as an approximate date for Jephthah’s activities, this would place the taking of the Transjordan under Moses (Numbers 21) around 1400 BC, about 40 years after the departure from Egypt.’, Hawkins, ‘Propositions For Evangelical Acceptance Of A Late-Date Exodus-Conquest:  Biblical Data And The Royal Scarabs From Mt. Ebal’, Journal of the Evangelical Theological Society, 50/1 (2007), 33.

[10] Exodus 1:11 So they put foremen over the Israelites to oppress them with hard labor. As a result they built Pithom and Rameses as store cities for Pharaoh.

[11] Harrison, ‘Exodus, The’, in Elwell & Beitzel, Baker Encyclopedia of the Bible (1988), 743.

[12] ‘Garstang identified several levels of debris there, indicating that the city had been rebuilt a number of times. He concluded that the one built about 1500 B.C. was the Jericho overthrown by Joshua’s forces (Jos 6). Garstang’s statement that Jericho had fallen before 1400 appeared to support the time frame of 1 Kings 6:1, and was received enthusiastically by supporters of the 15th-century B.C. date.’, ibid., p. 743.

[13] ‘Garstang’s discoveries at Jericho have been modified seriously by the subsequent work of another archaeologist, Kathleen Kenyon. She found no trace of Late Bronze Age walls, which indicates that the city Garstang thought to have been conquered by Joshua was considerably earlier than his time. Unfortunately, the mound has been so ravaged by erosion and human pillaging that it reveals almost nothing about the Jericho of Joshua’s day, and thus does not help to simplify matters.’, ibid., p. 744.

[14] Wood, ‘Did the Israelites Conquer Jericho? A New Look at the Archaeological Evidence’, Biblical Archaeology Review, 16/2 (March/April 1990), 44-58.

[15] Wood, ‘The Walls of Jericho’, Bible and Spade 12/2 (1999).

[16] ‘The 13th-century exodus-conquest theory was formulated by William F. Albright in the 1930s, based largely on Palestinian archaeological evidence, and promoted by him throughout his career.’, Wood, ‘The Rise and Fall of the 13th-Century Exodus-Conquest Theory’, Journal of the Evangelical Theological Society, 48/3 (2005), 473.

[17] ‘The Late Bronze Age (LB) was characterized by a problematic lack of fortified cities. The MB in Canaan, in sharp contrast, featured the massive and numerous walled cities that the books of Numbers and Joshua seem to suggest, and many of them were destroyed at the end of the period.’, Walton, ‘Exodus, Date of’, in Alexander & Baker, Dictionary of the Old Testament: Pentateuch (2003), 259.

[18] Ibid., p. 259.

[19] Harrison, ‘Exodus, The’, in Elwell & Beitzel, Baker Encyclopedia of the Bible (1988), 744.

[20] Ibid., p. 744.

h1

Failed prophecies?

July 3, 2014

Prophecies are consistently appealed to in the Bible as evidence of its divine origin. Several passages claimed by skeptics and atheists as examples of failed prophecies are examined here. They fall into the following categories: passages which are not formal predictive prophecies in the manner claimed, passages which are misinterpreted by the critic, and passages which were fulfilled in contradiction to the critic’s claims.

Passages which are applied typologically

Several passages in the Old Testament which have  traditionally been understood as prophetic of Christ, are objected to by critics as inapplicable or unfulfilled.

  1. How can ‘When he sins, I will correct him with the rod of men and with wounds inflicted by human beings’ in 2 Samuel 7:14 apply to Christ, when Christ never sinned?
  1. How can the servant of Isaiah 49 refer to Christ, when the context indicates it speaks explicitly of Israel?

A traditional answer has been that these prophecies have ‘dual fulfillments’, that they were prophecies fulfilled in part by Solomon or Israel, and fulfilled in part later in the life of Christ. This is actually only half true. In reality the prophecies had direct and full application to their immediate referent, and are applied typologically to Christ. That is, they are formal predictive prophecies about Solomon and Israel respectively, but parts of them find echoes in the life of Christ, the ‘son of God’ in a greater sense than both Solomon and Israel. Their application to Christ is not a claim to direct fulfillment of a formal prediction, but an illustration that certain promises made by God in earlier times have a relevance to the greater work of Christ which they foreshadow.

Passages which are misinterpreted

Certain prophecies criticized as unfulfilled have actually been misinterpreted.

  1. How can God’s words  ‘I, the LORD, promise: “David will never lack a successor to occupy the throne over the nation of Israel’ in Jeremiah 33:17 be said to have been fulfilled, when the Davidic monarchy ended with the Babylonian captivity?

The answer is that this prophecy was conditional, as was made clear to Solomon before Jeremiah’s time.

1 Kings 9:

You must serve me with integrity and sincerity, just as your father David did. Do everything I commanded and obey my rules and regulations.

Then I will allow your dynasty to rule over Israel permanently, just as I promised your father David, ‘You will not fail to have a successor on the throne of Israel.’

Solomon and his descendants failed to keep this covenant, and the Davidic monarchy consequently came to an end;[1] the New Testament writers identify Christ as its true successor.

  1. How can Jesus’ words ‘I tell you the truth, you will not finish going through all the towns of Israel before the Son of Man comes’ in Matthew 10:23 and ‘this generation will not pass away until all these things take place’ in Matthew 24:34 be considered anything but failed prophecies, when he did not return before the apostles had finished preaching to the towns of Israel, or before the generation of his time had passed away?

These passages do not speak of the return of Christ, but of the end which would come with the destruction of Jerusalem in the war of 66-70 CE.[2] [3]

  1. In Mark 8:38 Jesus says ‘there are some standing here who will not experience death before they see the kingdom of God come with power’, and in Matthew 16:28 Jesus says ‘there are some standing here who will not experience death before they see the Son of Man coming in his kingdom’, Jesus failed to return before his disciples died, making these false prophecies.

These passages refer to thetransfiguration (a vision of Jesus ‘coming in his kingdom’), which is described in both gospels as occurring shortly after Jesus spoke these words. Early Christians almost universally understood these passages as a reference to the transfiguration.[4]

It should be noted that these words of Jesus were transmitted by the gospel writers decades after Jesus’ ministry and after the disciples had died, indicating that they them as fulfilled prophecy. If these predictions had been understood as Jesus saying he would return before the disciples died, they would have been more likely to omit them completely.

  1. In 1 Corinthians 7:31 Paul says ‘the present shape of this world is passing away’, indicating that he, like other New Testament writers, believed they would see the return of Christ in their lifetime, but it never happened.

These words are not written as formal predictive prophecy, and as much as Paul may have believed at one time that Christ would return while he was still alive,  it is clear he and other writers such as Peter gradually understood that this would not be the case. In fact both Paul and Peter prepare other Christians for a lengthy wait before Jesus’ return. In 2 Thessalonians Paul makes it clear that Christ’s return is not imminent, and that it will not happen before certain specific events have taken place.[5]

2 Thessalonians 2:

1 Now regarding the arrival of our Lord Jesus Christ and our being gathered to be with him, we ask you, brothers and sisters,

2 not to be easily shaken from your composure or disturbed by any kind of spirit or message or letter allegedly from us, to the effect that the day of the Lord is already here.

3 Let no one deceive you in any way. For that day will not arrive until the rebellion comes and the man of lawlessness is revealed, the son of destruction.

Peter likewise prepares his readers for a lengthy wait, speaking of the ‘last days’ in the future, and warning believers they will be mocked for their patience at that time.

2 Peter 3:

3 Above all, understand this: In the last days blatant scoffers will come, being propelled by their own evil urges

4 and saying, “Where is his promised return? For ever since our ancestors died, all things have continued as they were from the beginning of creation.”

Passages which were fulfilled

Certain prophecies have actually been fulfilled, despite claims to the contrary.

  1. The prophecy of Isaiah 17 claims Damascus would be ‘a heap of ruins (verse 1), and that ‘Damascus will lose its kingdom’ (verse 23), but Damascus is a thriving city today.

This prophecy made by Isaiah some time around 740 BCE, was fulfilled when the Assyrian king Tiglath-pileser III destroyed Damascus, which was at that time not a mere city but a thriving Aramean kingdom.[6] Isaiah prophesied Damascus would be a ‘heap of ruins’, and Tiglath-pileser III’s own record of his conquest proves this is what happened, boasting of his extensive destruction of Damascus; ‘591 cities of the 16 districts of Damascus I destroyed like mounds of ruins after the Deluge’. [7] Isaiah’s prediction that ‘Damascus will lose its kingdom’ was fulfilled by Tiglath-pileser III’s annexation of Damascus and all its territories, and the deportation of  many of its inhabitants.[8] [9]

  1. The prophecy of Tyre’s destruction in Ezekiel 26 failed to come true, and Ezekiel’s own words in Ezekiel 29:18-19 (especially that Nebuchadnezzar II and his army ‘received no wages from Tyre for the work he carried out against it’ , verse 28), prove he knew the prophecy failed.

This criticism overlooks the fact that Ezekiel’s commentary in Ezekiel 29:18-19 is certainly in response to criticisms that his earlier prophecy was not completely successful; in other words, it proves at minimum that Ezekiel’s earlier prediction that Nebuchadnezzar II would attack Tyre was not only made before the event, but also came true. This is not a good start for a critic of prophetic fulfillment. Subsequent to Ezekiel’s successful prediction, his political enemies attempted to discredit him by claiming Nebuchadnezzar’s army had failed to  vindicate the prophecy.[10] Such claims were unwarranted, for ‘the siege was successful and Tyre did pass into Babylonian control’.[11]

Since Nebuchadnezzar II failed to destroy Tyre utterly, some commentators state that since Ezekiel’s original prophecy had predicted ‘many nations’ would be brought against Tyre (‘I will bring up many nations against you’, Ezekiel 26:3), of which Nebuchadnezzar II’s campaign was only the first. It is claimed that this is supported by the alternating statements of what ‘he’ (Nebuchadnezzar II), and ‘they’ (subsequent nations), would do to Tyre, especially Alexander the Great.

However, this view has been criticized as an improbable reading of the Hebrew text.[12] Instead it should be recognized that the first section of the prophecy (verses 1-6), is a self-contained unit predicting the coming of ‘many nations’ against Tyre, and thus not restricted to Nebuchadnezzar II (whose campaign is described from verses 7-13);[13] the complete destruction of the city as predicted by Ezekiel was fulfilled by Alexander.

The objection that the prophecy’s description of an attack against a mainland city (which Nebuchadnezzar attacked), and therefore does not describe the destruction of the island (which Alexander attacked), is misguided. [14] The prophecy uses the standard conquest language of the Ancient Near East; when Esarhaddon of Assyria attacked the island city he still described it in terms of a land battle, even to the point of describing trenches being dug (impossible in an island siege).[15]

  1. The prophecies against Egypt in Ezekiel 30 and Isaiah 19 failed to come true; Nebuchadnezzar did not invade and conquer Egypt as predicted.

First it should be noted that it is acknowledged even by modern critics that the prophecy was given before the event it describes took place.[16] Consequently, attempts to reduce the accuracy of the prediction by claiming it was insufficiently fulfilled are demonstrably motivated by the desire to avoid the fact that a successful prophecy actually happened. The best a critic can do in the face of the fact that the text contains a prophecy indisputably before the event to which it obviously refers, is to claim that the prophecy wasn’t fulfilled sufficiently to be considered accurate. This is not intellectually honest.

In fact, the prophecy was fulfilled by Nebuchadnezzar II’s war against Egypt in 586 BCE, recorded in a fragmentary Babylonian text.

‘. . . [in] the 37th year, Nebuchadnezzar, king of Bab[ylon] mar[ched against] Egypt (Mi-sir) to deliver a battle. [Ama]sis (text: [ . . . ]-a(?)-su)y of Egypt, [called up his a]rm[y] . . . [ . . . ]\u from the town Pufu-Iaman . . . distant regions which (are situated on islands) amidst the sea . . . many . . . which/who (are) in Egypt . . . [car]rying weapons, horses and [chariot]s . . . he called up to assist him and . . . did [ . . . ] in front of him . . . he put his trust. . . (only the first signs at the beginning and the end of the following 7 or 8 lines are legible).’[17]

The success of Nebuchadnezzar’s campaign is acknowledged by secular historians, on the basis of several lines of evidence.[18] Firstly, and rather ironically, some historians believe the prophecy of Ezekiel was written after the event, precisely because it is so accurate.

‘First of all, both Ezekiel and Jeremiah prophesied that he would do so; and since most of these “prophecies” were written in retrospect, or at least gained popular currency only after having been proved correct, we may be fairly certain that the prophesied invasion and defeat of Egypt actually took place.’[19]

Two other sources are the Biblical text describing Jewish refugees moved from Egypt to Babylon, and the record of Josephus indicating Nebuchadnezzar defeated Egypt.

‘Secondly, the biblical sources say that Nebuchadrezzar was able to remove the Jewish refugees in Egypt to Babylon. He could not of course have done so unless he had entered and subjugated the country. Thirdly, Josephus tells us that he conquered Egypt. We are informed that four years after the fall of Tyre, Nebuchadrezzar invaded the country and put its King Uaphris to death, installing a creature of his own upon the vacant throne.’[20]

Another source is the presence in Egypt of artifacts belonging to Nebuchadnezzar II, demonstrating he invaded and established himself there.

‘Fourthly, and most importantly, artifacts of Nebuchadrezzar have actually been discovered in Egypt. These are “three cylinders of terra-cotta bearing an inscription of Nebuchadnezzar, an ordinary text referring to his constructions in Babylon … These were said to come from the Isthmus of Suez, and they apparently belong to some place where Nebuchadrezzar had ‘set up his throne’ and ‘spread his royal pavilion.’ As he only passed along the Syrian road, and Daphnae would be the only stopping place on that road in the region of the isthmus, all the inferences point to these having come from Defenneh, and being the memorials of establishment there.”’ [21]

This also fulfills the prophecy of Jeremiah 43:10 that Nebuchadnezzar would ‘pitch his royal tent’ in Tahpanhes in Egypt.[22] [23] The scholarly  conclusion from these sources is ‘There can be little doubt; Nebuchadrezzar entered and conquered Egypt.’ [24]

  1. Although Joshua 3:10 says God would ‘truly drive out before you the Canaanites, Hittites, Hivites, Perizzites, Girgashites, Amorites, and Jebusite’, 1 Kings 9:20 says ‘several non-Israelite peoples were left in the land after the conquest of Joshua’.

Joshua’s words are a repetition of the words of Moses, which made it clear that God’s driving out of the  inhabitants of Canaan was conditional on the Hebrews maintaining their obedience to God.

Deuteronomy 4:

1 Now, Israel, pay attention to the statutes and ordinances I am about to teach you, so that you might live and go on to enter and take possession of the land that the LORD, the God of your ancestors, is giving you.

Deuteronomy 7:

12 If you obey these ordinances and are careful to do them, the LORD your God will faithfully keep covenant with you as he promised your ancestors.

Deuteronomy 8:

1 You must keep carefully all these commandments I am giving you today so that you may live, increase in number, and go in and occupy the land that the LORD promised to your ancestors.

  1. The gospels describe Jesus predicting the destruction of Jerusalem and the Temple, but this is not an accurate prediction because the gospels were written after the event; even if they had been written before the event, such a war was obviously going to happen anyway, so it is not evidence of an accurate prophecy.

The first point to note about this objection (which can be found made by a range of atheists and skeptics), is that it is intellectually dishonest; it tries to argue that the prophecy isn’t true because it was written after the event, but also argues that even if it had been made before the event it still wouldn’t count as a fulfilled prophecy. Such an argument is not evidence based, and demonstrates that the person making the argument is not really interested in the facts.

The second point to note about this objection is that although the date of Mark’s gospel is still an open question in current scholarship, dates proposed typically fall between 65 and 75 CE.[25] In fact recently strong arguments have been made for a much earlier date. The secular scholar Mark Crossley argues for a date ‘before the late forties’,[26] at least 20 years before the destruction of Jerusalem, and the secular scholar Maurice Crossley argues ‘a date c. 40 CE must be regarded as highly probable’.[27]

It is clear that many scholars (even non-religious scholars), are prepared to accept that Mark’s gospel (which contains a prophecy of the destruction of Jerusalem and the Temple), was indeed written before the events it predicts. Rejecting this possibility out of hand is therefore intellectually dishonest, especially when no attempt is made to address the significant body of evidence indicating that Jesus’ prediction in Mark was indeed made before the event.

The third point to note is that there is no evidence at all to suggest that the possibility of the destruction of Jerusalem and the Temple by the Romans was so probable that it would have appeared obvious to people in Jesus’ own day, or even 30 years later (63-65 CE). In fact construction on the Temple had already been carried out for nearly 50 years by the time of Jesus’ ministry (John 2:20), and it was not even completed until shortly before the Romans destroyed it.[28][29]

It is difficult to demonstrate that anyone seeing the ongoing construction of the Temple in Jesus’ day would have concluded that the Romans would destroy it over 30 years later. It is likewise difficult to demonstrate that anyone seeing the construction continuing in 63 CE would have concluded that the Romans would destroy both the Temple and the entire city in just a few more years.

The fact is that the explosive events which led to the destruction of Jerusalem in 70 CE were not predicted by anyone outside the gospels, and even Jesus’ own words make it clear he expects skepticism on the part of his audience. There is no evidence in any of the relevant Roman historical sources who wrote in detail about the events of the first century (such as Suetonius, Tacitus, and Appian), nor in Josephus (who was both a historian, and a general on the Jewish side of the war), that anyone was expecting such a war before it took place, still less the complete destruction of Jerusalem and the Temple. There is excellent evidence therefore that Jesus’ prophecy was made well before the events they predicted successfully, and it is indisputable that these events were not foreseen or expected by anyone else.

Conclusion

Skeptical claims that Bible prophecies have not been fulfilled should be taken seriously; it is a fact that some Bible prophecies are impossible to verify due to a lack of available information, even if there is no evidence proving they did not come to pass. However, when such criticisms are made it is important to identify whether or not the arguments made are evidence based, demonstrate a knowledge of and engagement with the relevant scholarly literature, and are intellectually honest.

In particular, such arguments must prove that they have interpreted the prophecy as it was originally intended, and must provide substantial objections to the relevant counter-arguments by scholars who make the case that the prophecy was in fact fulfilled. In turn, defenses of these prophecies must exercise intellectual honesty in acknowledging problems where they genuinely exist, and must cite, discuss, and be supported by the relevant scholarly literature, in order to be credible.

____________________________________

[1] ‘But perhaps the promise to the house of David is not all that unilaterally unconditional, and perhaps the dynastic oracle here is firmly set in the Sinaitic covenant (Eslinger 1994). We can agree with Brueggemann (1990: 259) when he says, “While the covenantal ‘if’ is silenced in this theology, it has not been nullified.” This is particularly true when the Lord speaks of David’s son: “When he commits iniquity, I will punish him with a rod such as mortals use, with blows inflicted by human beings” (v. 14b).’, Victor P. Hamilton, Handbook on the Historical Books (Grand Rapids, MI: Baker Academic, 2001), 317–318.

[2]Now that we have seen that the reference is to the destruction of the temple, which did as a matter of fact take place some 40 years later while many of Jesus’ contemporaries must have been still alive, all such contrived renderings may be laid to rest. This verse refers to the same time-scale as 16:28 (which was also concerned with the fulfillment of Dan 7:13–14): “some of those standing here will certainly not taste death before …” (cf. also 10:23, with the same Daniel reference: “you will not go through all the towns of Israel before …”).’, R. T. France, The Gospel of Matthew (The New International Commentary on the New Testament; Grand Rapids, MI: Wm. B. Eerdmans Publication Co., 2007), 930.

[3]34 The πάντα ταῦτα, “all these things,” of this verse can include no more than the same phrase in the preceding verse and thus cannot include the coming of the Son of Man (so too Blomberg). The phrase refers not only to general marks of the interim period such as tribulation, distress, pseudo-messiahs, and false prophets but specifically, and dramatically, to the desecration of the temple and the destruction of Jerusalem (cf. vv 15–22). As in the other imminence sayings (cf. 16:28; 10:23; 23:36), all of which like the present logion are prefaced by the emphatic ἀμὴν λέγω ὑμῖν, “truly I tell you,” formula, the main point is that the fall of Jerusalem was to be experienced by that generation (pace Kidder), those listening there and then to the teaching of Jesus (ἡ γενεὰ αὕτη, “this generation,” is used consistently in the Gospel to refer to Jesus’ contemporaries; cf. 11:16; 12:41–42, 45; 23:36).’ Donald A. Hagner, Matthew 14–28 (vol. 33B; Word Biblical Commentary; Dallas: Word, Incorporated, 1998), 715.

[4]‘The most widespread interpretation in the Eastern and Western church related the saying to the transfiguration whereby then “some” referred to Peter, James, and John.’, Ulrich Luz, Matthew: a Commentary (ed. Helmut Koester; Hermeneia—a Critical and Historical Commentary on the Bible; Minneapolis, MN: Augsburg, 2001), 386; Luz disagrees with this interpretation, but acknowledges it was the most common in the early church.

[5] ‘Mearns (“Development”) also takes the reference to be to 1 Thessalonians, but argues that Paul had changed his mind about the suddenness of the Day of the Lord since writing that letter and is now correcting perfectly reasonable inferences that the readers might have drawn from it.’, F. F. Bruce, 1 and 2 Thessalonians (vol. 45; Word Biblical Commentary; Dallas: Word, Incorporated, 1998), 164.

[6] ‘The Assyrian invaded northern Israel (2 K. 15:29) and then besieged and destroyed Damascus as an Aramean kingdom in 732, killing Rezin (2 K. 16:9). Assyrian records tell of 591 towns of the “16 districts of Aram” destroyed “like mounds left by a flood” (ARAB, I § 777).’, M. F. Unger, “Damascus,” ed. Geoffrey W. Bromiley, The International Standard Bible Encyclopedia, Revised (Wm. B. Eerdmans, 1979–1988), 854.

[7] ‘12′ […] orchards without number I cut down; I did not leave a single one. 13′ … the town of …]hadara, the home of the dynasty of Rezin of Damascus, 14′ [the pl]ace where he was born, I surrounded and captured. 800 people with their possessions 15′ their cattle (and) their sheep I took as spoil. 750 captives from the cities of Kuruṣṣa 16′ (and) Sama, 550 captives from Metuna I took, 591 cities 17′ of the 16 districts of Damascus I destroyed like mounds of ruins after the Deluge.’, Lester L. Grabbe, “The Kingdom of Israel from Omri to the Fall of Samaria: If We Only Had the Bible …,” in Ahab Agonistes: The Rise and Fall of the Omri Dynasty (ed. Lester L. Grabbe; London: T&T Clark, 2007), 79.

[8] ‘Finally, the Assyrians had had enough of the rebellious behaviour of Damascus, the last Aramaean stronghold in Hatti. Damascus and its cities were conquered and turned into Assyrian provinces (Tadmor 1994: 79–81). A part of the population was deported (Grayson 1991/2000: 77–8; Dion 1997: 215–16; Sader 1987: 250–1; Weippert 1987: 99).’, Hans M. Barstad, “Can Prophetic Texts Be Dated? Amos 1–2 as an Example,” in Ahab Agonistes: The Rise and Fall of the Omri Dynasty (ed. Lester L. Grabbe; London: T&T Clark, 2007), 33.

[9] ‘At the close of the Syro-Ephraimitic War, Tiglath-pileser took several actions that form the background of this text. He killed Rezin, destroyed Damascus, and annexed all territory controlled by Damascus into the Assyrian provincial system.‘, Brad E. Kelle, “What’s in a Name? Neo-Assyrian Designations for the Northern Kingdom and Their Implications for Israelite History and Biblical Interpretation,” ed. Gail R. O’Day, Journal of Biblical Literature 121 (2002): 639., Gail R. O’Day, ed., Journal of Biblical Literature 121 (2002): 659.

[10] ‘The objective reason for the oracle is supplied in v 18. However, the more immediate agenda is implied by v 21ab: Ezekiel was being criticized by his Jewish contemporaries for the lack of precise fulfillment of his oracles against Tyre.’. Leslie C. Allen, Ezekiel 20–48 (vol. 29; Word Biblical Commentary; Dallas: Word, Incorporated, 1998), 109.

[11] ‘It was to some extent a carping criticism: the siege was successful and Tyre did pass into Babylonian control. In a list of royal hostages at Nebuchadnezzar’s court, to be dated about 570 B.C., the king of Tyre has the initial place (ANET 308a; Katzenstein, History of Tyre 326). About 564 B.C. Baal, Ethbaal’s successor as king of Tyre, was replaced by a Babylonian High Commissioner (Katzenstein, History 332–33; cf. Unger, ZAW 44 [1926] 314–17). Any prophet might have been glad to chalk it up as a vindication of his or her prediction, despite Nebuchadnezzar’s non-destruction of Tyre.’, ibid., p. 109.

[12] ‘However, the proposed distinction between the “many nations” of verse 3 and Nebuchadnezzar’s army seems overly subtle, in light of the reference to Nebuchadnezzar as “king of kings” (v. 7) and the multiethnic nature of his army. Nebuchadnezzar is the focal point of verses 7–11, but the actions described are those of an army. The subject of the plural forms in verse 12 is most naturally understood as the collective “army” (Heb. ‘am) of verse 7, which in turn can be seen as comprised of the “many nations” mentioned in verse 3 (see also the reference to “nations” in v. 5).’, Robert B. Chisholm Jr., Handbook on the Prophets: Isaiah, Jeremiah, Lamentations, Ezekiel, Daniel, Minor Prophets (Grand Rapids, MI: Baker Academic, 2002), 268.

[13]Tyre’s fall appears in the first two prophecies, the second picking up phrases from the first and adding further details, and the remaining two prophecies describe the bewailing and then the entombment of the fallen one.’, Ronald M. Hals, Ezekiel (vol. 19; The Forms of the Old Testament Literature; Grand Rapids, MI: William B. Eerdmans Publishing Company, 1989), 188.

[14]Taking the standard siege imagery too literally, some scholars have concluded that the passage must come out of a setting different from Nebuchadrezzar’s time, seeing in vv. 9–11 a battle song about Alexander’s conquest of Tyre, or supposing that the actual reference of the passage was to a conquest of “old Tyre” on the adjacent mainland. (See the references in Zimmerli, Ezekiel 2, 37.) The proper recognition of the typicality of the imagery involved renders such literalistic hypotheses unnecessary.’, Ronald M. Hals, Ezekiel (vol. 19; The Forms of the Old Testament Literature; Grand Rapids, MI: William B. Eerdmans Publishing Company, 1989), 189.

[15] ‘The infiltration of characteristics typical of a description of a mainland siege into the description of the siege of the island city of Tyre can already be observed in the Assyrian royal inscriptions, when Esarhaddon there reports of the construction of trenches (ḫalṣē) against Tyre.’, Walther Zimmerli, Frank Moore Cross, and Klaus Baltzer, Ezekiel: a Commentary on the Book of the Prophet Ezekiel (Hermeneia—a Critical and Historical Commentary on the Bible; Philadelphia: Fortress Press, 1979–), 37; a footnote adds ‘This also disposes of Wiener’s theory that this passage deals solely with the conquest of the mainland city of old Tyre’.

[16] ‘The prophecy against Egypt does not seem to have come to pass in every detail either, but the book was probably completed and its authority established by the time this became clear.’, Thomas Renz, The Rhetorical Function of the Book of Ezekiel (Leiden: Brill, 2002), 98.

[17] J. B. Pritchard (ed.), Ancient Near Eastern Texts (3rd ed. with supplement. Princeton: Princeton UP, rev. 1969), 308.

[18] ‘That Nebuchadrezzar actually conquered Egypt is suggested by a number of very powerful pieces of evidence’, Emmet John Sweeney, The Ramessides, Medes, and Persians, Ages In Alignment Series, volume 4 (Algora Publishing, 2008), 153; ‘Nebuchadrezzar’ is the more accurate transliteration of the name ‘Nebuchadnezzar’.

[19] Ibid., p. 153.

[20] Ibid., p. 153.

[21] Ibid., p. 153.

[22] Jeremiah 43: 8 At Tahpanhes the LORD spoke to Jeremiah. 9 “Take some large stones and bury them in the mortar of the clay pavement at the entrance of Pharaoh’s residence here in Tahpanhes. Do it while the people of Judah present there are watching. 10 Then tell them, ‘The LORD God of Israel who rules over all says, “I will bring my servant King Nebuchadnezzar of Babylon. I will set his throne over these stones which I have buried. He will pitch his royal tent over them.

[23] ‘In short, the prophecy of Jeremiah that the king of Babylon would spread his royal pavilion at the entrance of the pharaoh’s house in Tahpanheth (Daphnae) was fulfilled.’, ibid., p. 153.

[24] Ibid., p. 153.

[25] ‘While scholars differ over the precise year, a date between 65 and 75 CE is accepted by a wide variety of scholars of very different ideological persuasions.’, James G. Crossley, The Date of Mark’s Gospel: Insight from the Law in Earliest Christianity (vol. 266; Journal for the Study of the New Testament Supplement Series; London; New York: T&T Clark International, 2004), 1.

[26] ‘This now becomes an argument of powerful collective weight for Mark to have been written before the late forties and if this is combined with the analysis of Mark 13 in Chapter 2 it is unlikely that it was written no earlier than the mid to late thirties.’, ibid., p. 208.

[27] Maurice Casey, Aramaic Sources of Mark’s Gospel (vol. 102;  Society for New Testament Studies; Cambridge University Press, 1998), 260.

[28] ‘According to Josephus,17 the Herodian temple was begun in 20/19 BCE; it was completed shortly before the war with Rome.18 If this scene may be used to date the events,19 it would have taken place on Passover of the year 28.’, Ernst Haenchen, Robert Walter Funk, and Ulrich Busse, John: a Commentary on the Gospel of John (Hermeneia—a Critical and Historical Commentary on the Bible; Philadelphia: Fortress Press, 1984), 184.

[29] ‘Work was still going on at his [Herod’s] death, and for that matter, for long after. The Temple was not completed until A.D. 63.’, Leon Morris, The Gospel According to John (The New International Commentary on the New Testament; Grand Rapids, MI: Wm. B. Eerdmans Publishing Co., 1995), 176.

h1

Living On The Edge: challenges to faith

September 1, 2013

Today Christians in the Western world are typically living in a post-Christian society. Christian beliefs are met with skepticism, and people see little reason to believe. Christians are confronted with daily challenges to their faith, and often struggle to understand the relevance of Christianity to modern life.

The book ‘Living On The Edge: challenges to faith‘ (due to be printed in November 2013), addresses those concerns. For an overview of the book, click here.

h1

Is the Bible’s chronology of the kings of Israel accurate?

July 30, 2011

The Challenge

In the late 19th century, critical scholar Julius Wellhausen claimed the Biblical chronology of the kings of Israel was a literary invention for religious purposes, which had been edited and revised several times from a variety of different sources, rather than a genuine historical record.[1]

For the next 70 years, critical scholars continued to treat the chronology as historically worthless and irreconcilable.[2]

The Facts

In 1951, Biblical scholar Edwin Thiele published ‘The Mysterious Numbers of the Hebrew Kings’, a harmonization of the Biblical record of the kings of Israel (originally as a doctoral dissertation). By the time of the second edition (slightly revised), it was recognized that Thiele’s work was a significant breakthrough in establishing the historical validity of the Biblical chronology.[3]

Reception

Though criticisms have been made of Thiele’s chronology,[4] [5] [6] [7] its value and general validity have been acknowledged widely.[8] [9] [10] [11] [12]

It remains the typical starting point for study of the chronology of the kings of Israel and Judah[13] [14] with few modifications,[15] [16]   and has been applied successfully in other fields of Ancient Near East study, such as the chronologies of Assyria and Babylon.[17]

The reliability of the chronologies in 1-2 Kings has been supported by archaeological evidence; Grabbe notes that the chronology in these books ‘agrees with what can be gleaned from extra-biblical sources’, and that ‘even if we had no external sources we could have reasonable confidence in the biblical sequence’.[18]


[1] ‘That a process of alteration and improvement of the chronology was busily carried on in later times, we see from the added synchronisms of the kings of Israel and Judah,’, Wellhausen, ‘Prolegomena to the History of Israel’, p. 278 (1885).

[2] ‘Driver remarked that, “the length of the reigns of the various kings is not the same according to the traditional and the synchronistic figures. Since, however, it is clear on various grounds that these synchronisms are not original, any attempt to base a chronological scheme on them may be disregarded.” Kittel stated his view that, “Wellhausen has shown, by convincing reasons, that the synchronisms within the Book of Kings cannot possibly rest on ancient tradition, but are on the contrary simply the products of artificial reckoning. . . The Israelitish numbers and the parallel numbers referring to Judah do not agree at the points at which we are able to compare them.” Robinson also was impressed by Wellhausen’s evaluation: “Wellhausen is surely right in believing that the synchronisms in Kings are worthless, being merely a late compilation from the actual figures given.” * R. H. Pfeiffer’s opinion was that, “The chronology based on the synchronisms is of course less reliable than the one based on the regnal periods, since the synchronisms were figured from the regnal periods. Neither chronology is wholly accurate . . . In spite of these discrepancies, inaccuracies, and errors, the chronology of Kings is not fantastic.” 5 J. F. McCurdy expressed himself to the effect that, “Many of the numbers given, especially the synchronisms, are erroneous, as is proved by the fact that no attempt to harmonize the two series has been successful . . . Startling inconsistencies are also found where the several synchronisms for the same king are worked out.” K. Marti gave his observation: “The synchronistic notes betray their character as ‘subjective additions of the Epitomator.’ It is clear, to begin with, that this noting of synchronisms was not in actual use during the existence of the two kingdoms. . . Almost along the whole line, the discrepancy between synchronisms and years of reign is incurable.” C. H. Gordon observed: “The numerical errors in the Books of Kings have defied every attempt to ungarble them. Those errors are largely the creation of the editors who set out to write a synchronistic history of Judah and Israel, using as sources two sets of unrelated court chronicles. Combining two elaborate sets of figures was not an easy task. But even with due regard for the difficulties involved, the editors did not execute the synchronisms skillfully.”’, Thiele, ‘Synchronisms of the Hebrew Kings – A Re-evaluation: I’, Andrews University Seminary Studies, pp. 14-125 (1), 1963.

[3]A marked advance in biblical scholarship was made in the publication of The Mysterious Numbers of the Hebrew Kings, U. of Chicago Press, 1951, by Dr. E. R. Thiele. In his revised edition in 1965 (Eerdmans Pub. Co., Grand Rapids, Mich.), Dr. Thiele asserts the soundness of his basic thesis and conclusions as confirmed by scholars since his first edition.’, editorial, Journal of the Evangelical Theological Society (9.1.60), 1966.

[4] ‘Thiele’s view contains positive elements, but it also poses numerous difficulties. He incorrectly understood the annals of Tiglath-pileser III, and his determination that Menahem died in 742 contradicts the testimonies of the contemporaneous Assyrian inscriptions.10 In his desire to resolve the discrepancies between the data in the Book of Kings, Thiele was forced to make improbable suppositions. He assumed that the system of counting the years of reign changed every few generations, or even after a few decades. This is improbable, and cannot be proved. Similarly, he presumes that the Northern and Southern Kingdoms numbered their years both by the local count and by that practiced by the other kingdom, also for short periods, while this practice ceased in other periods. Thiele even went so far as to assume that while this practice had fallen into disuse, there were scribes who continued to calculate the years in accordance with it. There is no basis for Thiele’s statement that his conjectures are correct because he succeeded in reconciling most of the data in the Book of Kings, since his assumptions regarding Biblical chronological principles are derived from the chronological data themselves, whose reliability is unclear.’, Galil, ‘The Chronology of the Kings of Israel and Judah’, p. 4 (1996).

[5]but his harmonizing approach has not gone unchallenged, especially because of the many shifts in the basis of reckoning dates that it requires (e.g., Jepsen 1968: 34–35)—shifts which were unlikely in actual practice. The numerous extrabiblical synchronisms he invokes do not always reflect the latest refinements in Assyriological research (cf. E.2.f below). In many cases, he posits an undocumented event in order to save a biblical datum (e.g., the circumstances surrounding the appointment of Jeroboam II as coregent; Thiele 1983: 109)’, Cogan, ‘Chronology (Hebrew Bible)’, in Freedman, (ed.), ’The Anchor Yale Bible Dictionary’, volume 1, p. 1006 (1996)’, Freedman, DN (1996).

[6]  ‘Despite that fact of scholarly dedication, neither Thiele’s carefully argued University of Chicago dissertation, nor anyone else’s, has achieved as yet universal acceptance.’, Kaiser, ‘A History of Israel: From the bronze age through the Jewish Wars’, p. 293 (1998).

[7]Not all scholars are convinced by this solution, and commentators on the prophetic books often accept that dates can only be approximate.’, McConville, ‘Exploring the Old Testament, Volume 4: The Prophets’, p. viii (2002).

[8] ‘The chronology most widely accepted today is one based on the meticulous study by Thiele. Wiseman, ‘1 and 2 Kings’, Tyndale Old Testament Commentaries, p. 27 (1993).

[9]Increasingly his chronological scheme has come to dominate the majority of scholarly works and it is unlikely that his system can ever be overthrown without altering some well-established dates in Near Eastern history, for Thiele’s chronology is now inextricably locked into the chronology of the Near East.’, McFall, ‘A Translation Guide to the Chronological Data in Kings and Chronicles’, Bibliotheca Sacra (148.589.42-43), 1996.

[10] ‘Thiele’s system of chronology has been well received over the past 40 years and is now accepted as the basis for Israel’s chronology in a growing number of standard scholarly works.’, ibid., p. 42; see for example: Mitchell, ‘Israel and Judah until the Revolt of Jehu (931-841 B.C.)’, Cambridge Ancient History, volume 3, part 1, p. 445 (1982); Finegan, ‘Handbook of Biblical Chronology’, p. 249 (rev. ed.1998); Hess, ‘Chronology (Old Testament)’, in Porter (ed.), ‘Dictionary of Biblical Criticism and Interpretation’, p. 55 (2007).

[11]Thiele’s chronology is fast becoming the consensus view among Old Testament scholars, if it has not already reached that point.’, McFall, ‘The Chronology of Saul and David’, Journal of the Evangelical Theological Society (53.101.215), 2010.

[12]Thiele’s chronology (which differs from that of the present paper in only a few places) won the respect of historians because its dates agree with the following dates in Assyrian and Babylonian history: the Battle of Qarqar in 853 bc; the tribute of Jehu to Shalmaneser III in 841 bc; the capture of Samaria by Shalmaneser V in 723 bc; the invasion of Sennacherib in 701 bc; the Battle of Carchemish in 605 bc; and the first capture of Jerusalem by Nebuchadnezzar in 597 bc.’, Young, ‘Tables of Reign Lengths From the Hebrew Court Recorders’, Journal of the Evangelical Theological Society (48.2.232), 2005.

[13] ‘Thiele’s work has become a cornerstone of much recent chronological discussion (cf. De Vries IDB 1: 580–99; IDBSup: 161–66);’, Cogan, ‘Chronology (Hebrew Bible)’, in Freedman, (ed.), ’The Anchor Yale Bible Dictionary’, volume 1, p. 1006 (1996)’, Freedman, DN (1996).

[14] ‘Although some would prefer to see transmission errors where Thiele invokes the above principles, his chronology remains the starting point for all discussions of the debate.’, Hess, ‘Chronology (Old Testament)’, in Porter (ed.), ‘Dictionary of Biblical Criticism and Interpretation’, p. 55 (2007).

[15] ‘After 40 years Thiele’s chronology has not been significantly altered or proved to be false in any major area except in the matter of Hezekiah’s coregency.’, McFall, ‘A Translation Guide to the Chronological Data in Kings and Chronicles’, Bibliotheca Sacra (148.589.42), 1996.

[16]It remained then for others to complete the application of principles that Thiele used elsewhere, thereby providing a chronology for the eighth-century kings of Judah that is in complete harmony with the reign lengths and synchronisms given in 2 Kings and 2 Chronicles. The most thorough work in this regard was Leslie McFall’s 1991 article in Bibliotheca Sacra.22 McFall made his way through the reign lengths and synchronisms of Kings and Chronicles, and using an exact notation that indicated whether the years were being measured according to Judah’s Tishri years or Israel’s Nisan years, he was able to produce a chronology for the divided monarchies that was consistent with all the scriptural texts chosen.’, ibid., pp. 105-106.

[17] ‘In a 1996 article, Kenneth Strand wrote, “What has generally not been given due notice is the effect that Thiele’s clarification of the Hebrew chronology of this period of history has had in furnishing a corrective for various dates in ancient Assyrian and Babylonian history.”28 The purpose of Strand’s article was to show that Thiele’s methodology accomplished more than just producing a coherent chronology from scriptural data. His chronology, once produced, proved useful in settling some troublesome problems in Assyrian and Babylonian history.’, Young, ‘Inductive And Deductive Methods As Applied To OT Chronology’, Master’s Seminary Journal (18.1.112-113), 2007.

[18] ‘Grabbe suggests that the names and sequence of kings in Israel and Judah, and their approximate chronological placement, agrees with what can be gleaned from extra-biblical sources. To this extent the biblical framework (meaning primarily 1 and 2 Kings) is reliable: even if we had no external sources we could have reasonable confidence in the biblical sequence of Jeroboam I, Nadab, Baasha, Elah, Omri, Ahab, Jehu, etc. in Samaria, and David, Solomon, Rehoboam, Abijam, Asa, Jehoshaphat, etc. in Jerusalem, along with their interrelationships. Beyond that it starts to get more and more tricky, with decreasing reliability in the biblical narrative as the detail increases (this is a general statement, and there are sometimes exceptions in specific instances).’, Grabbe, ‘Reflections on the Discussion’, Grabbe (ed.), ‘Ahab Agonistes: The Rise and Fall of the Omri Dynasty’, p. 337 (2007).

h1

Is Solomon’s wealth a literary fiction?

April 30, 2011

The Challenge

The Biblical account of Solomon’s wealth has been described as unrealistic, in standard critical commentaries.[1] Many scholars are sceptical, [2] [3] [4] though some express their doubts cautiously.[5] [6]

Ancient Uses of Gold

Ancient uses of gold for construction which are analogous to Solomon’s include the tomb of Tutankhamen,[7] extensive use of gold plating on buildings in the reign of Tuthmosis III,[8] massive gold use on buildings of the Egyptian New Kingdom era,[9] and the same kind of gold usage in Babylonia and Assyria.[10] Millard also points out that items described as ‘of gold’ were not always solid gold; often they were covered in gold plate or gold leaf.[11]

Solomon’s Income

The Bible identifies ‘Ophir’ as one source of Solomon’s gold.[12] Although the location of Ophir is unknown, archaeological evidence identifies it as a source of gold.[13] Solomon’s income of 666 talents of gold in one year[14] [15]  is considered fictional by some commentators.[16]

Although this income is unique in Ancient Near East records,[17] the 120 talents of gold received by Solomon from Tyre[18] is matched and exceeded by gifts and tribute of gold from other Ancient Near East monarchs.[19] [20] [21] [22] [23]

The vast gold expenditure of pharaoh Orsokon I exceeded even Solomon’s, [24] and it is likely his wealth was the result of his father Sheshonq’s conquest of Solmon’s son Rehoboam.[25] [26] [27] [28]


[1]The gilding of the furnishings, as of the altar, is reasonable, but not that of the whole interior; cf. Stade, and Nowack, Arch., 2, 29, n. I.’, Montgomery, ‘A Critical and Exegetical Commentary on the Book of Kings’, p. 152 (1951).

[2]Such extravagant description appears to be a step forward in the process of exuberant imagination, continued by the Chronicler, for whose fancy even the 120-cubit high portico was overlaid with fine gold (2 Ch. 34ff.).’, ibid., p. 152.

[3] ‘Some have regarded this description as exaggerated.’, Hicks, ‘1 & 2 Chronicles’, College Press NIV Commentary, p. 306 (2001).

[4] ‘Some have questioned the authenticity of this description, labeling it unabashed exaggeration.’, Long, ‘1 & 2 Kings’, College Press NIV Commentary, p. 147 (2002).

[5]Despite all exaggerated accounts of Solomon’s wealth and commercial success, which were written to give him honor and prestige, there is an historical kernel in the reports of his wealth.’, Esler, ‘Ancient Israel: The Old Testament in its social context’, p. 105 (2006).

[6] ‘Evidently, we can not take the figures about Solomon’s mercantile activities and revenues given in the account at face value. They must have been fabulously exaggerated. Nevertheless, in Ishida’s assessment, which I share, “We can hardly deny the substantial historicity comprised in them” (p. 109).’, Corral, ‘Ezekiel’s Oracles Against Tyre: historical reality and motivations’, p. 112 (2002).

[7] ‘Here were many articles of furniture plated with sheets of gold, beaten and engraved, a wealth of elaborate golden jewellery, a golden dagger, the king’s gold mask, and, eclipsing all, his coffin of solid gold.7 Its weight is 110.4 kg (243 lbs). Particularly relevant for the present study are the shrines that stood in the tomb. There is a small wooden shrine (50 cms high, 26.6 cms wide, 32 cms deep, 19¾ x 10½ x 12¾ inches) made to hold a statue. Sheets of gold cover it entirely, within and without, embossed and engraved with scenes of the king’s life, magical figures, and inscriptions.’, Millard, ‘Solomon In All His Glory’, Bible and Spade (11.2-3-4.64-65), 1982.

[8] ‘In the Temple of the Sacred Boat at Karnak stood twelve columns erected by Tuthmosis III, about 1450 BC, each about 3½ metres high, designed to represent bundles of papyrus. Each was entirely covered with gold, fastened in slits cut at suitable points in the pattern. In another hall at Karnak were fourteen columns. Their design was similar, a papyrus stem, and they, too, were plated with gold from top to bottom. These pillars were larger; an inscription states that they were 31 cubits, that is 16.25 metres high (53 feet).’, ibid., p. 67.

[9] ‘Tuthmosis III (c. 1490–1436 BC) recorded his building of a shrine ‘plated with gold and silver’, and of a floor similarly made. Amenophis III in the next century decorated several structures in this way. Of one temple in honour of Amun at Thebes he claimed it was ‘plated with gold throughout, its floor is adorned with silver, all its portals with electrum’, while the temple at Soleb had the same treatment, except that ‘all its portals are of gold’. Ramesses II (c. 1297–1213 BC) provided his mortuary temple at Abydos with doors ‘mounted with copper and gilded with electrum’. Later in this period, Ramesses III (c. 1183–1152 BC) ornamented temples in exactly the same way. At Medinet Habu he constructed a shrine of gold with a pavement of silver, and doorposts of fine gold.’, ibid., p. 68.

[10] ‘Esarhaddon of Assyria (680–669 BC) told how he restored the shrine of his national god, Assur, and ‘coated the walls with gold as if with plaster’. His son Ashurbanipal claimed much the same, ‘I clad its walls with gold and silver’. In Babylon a century later Nebuchadnezzar recorded his enrichment of the shrines of his gods, ‘I clad (them) in gold, and made them bright as day’, and Nabonidus (555–539 BC) followed him, ‘I clad its walls with gold and silver, and made them shine like the sun’. The tradition stemmed from much earlier times in Babylonia, for Entemena of Lagash built a temple for his god ‘and covered it with gold and silver’ about 2400 BC.16’, ibid., pp. 68-69.

[11]While words like ‘a gold statue’ or ‘a gold bed’ in ancient documents should not be pressed to mean ‘made of solid gold throughout’ or ‘the purest gold’, they can be understood to mean ‘gold all over’, that is to say, nothing else could be seen.’, ibid., pp. 69-70.

[12] 1 Kings 9: 28 They sailed to Ophir, took from there four hundred twenty talents of gold, and then brought them to King Solomon.

[13]The expression “gold of Ophir” occurs not only in the Bible, but also on an eighth-century B.C. ostracon* found at Tell Qasile in Israel. That ostracon, while showing that the name was current to designate the origin or type of gold, throws no light on Ophir’s location.’, Millard, ‘Does the Bible Exaggerate King Solomon’s Golden Wealth?’, Biblical Archaeology Review (15.03), May/June 1989.

[14] 1 Kings 10:14 Solomon received 666 talents of gold per year; commentators are divided as to whether this represents an annual income, or the income of one particular year.

[15] ‘On the basis of these figures, Solomon’s gold can be computed as: 120 talents==3,960 kg==3.9 tons from Tyre, and the same from Sheba; 420 talents==13,860 kg==13.6 tons from Ophir; 666 talents==21,978 kg==21.6 tons in one year.’, Millard, ‘Solomon In All His Glory’, Bible and Spade (11.2-3-4.72), 1982.

[16] ‘Indeed, J.B. Pritchard argues that the narrative’s references to gold, pure gold and silver and its allusions to the respect which Solomon’s peers showed to him are ‘popular—even folkloristic’ elements of the history of Solomon’s age.’, Younger Jr, ‘The Figurative Aspect and the Contextual Method in the Evaluation of the Solomonic Empire: 1 Kings 1–11’, in Clines, Fowl & Porter (eds.), ‘The Bible in Three Dimensions: Essays in Celebration of Forty Years of Biblical Studies in the University of Sheffield’, p. 159 (1990).

[17]The only ancient text that reports the annual income of a powerful king in Old Testament times is the Hebrew Bible. In 1 Kings 10:14 the figure of 666 talents of gold (almost 25 U.S. tons) is given for Solomon. This may refer to a particular year, just as the 420 talents (15.75 U.S. tons) from Ophir refers to a particular source (1 Kings 10:11). Only two figures in ancient records approach the amount of 666 talents: the total of Pharaoh Osorkon’s gift to the gods and the amounts of treasure Alexander the Great found in Persia.’, Millard, ‘Does the Bible Exaggerate King Solomon’s Golden Wealth?’, Biblical Archaeology Review (15.03), May/June 1989.

[18] 1 Kings 9: 28 They sailed to Ophir, took from there four hundred twenty talents of gold, and then brought them to King Solomon.

[19] ‘We learn from firsthand sources that Metten II of Tyre (ca. 730) paid a tribute of 150 talents of gold to our old acquaintance Tiglath-pileser III of Assyria, while in turn his successor Sargon II (727-705) bestowed 154 talents of gold upon the Babylonian gods – about 6 tons in each case. Going back almost eight centuries, Tuthmosis III of Egypt presented about 13.5 tons (well over 200 talents) of gold in nuggets and rings to the god Amun in Thebes, plus an unknown amount more in a splendid array of gold vessels and cult implements. Worth almost a third of Solomon’s reputed annual gold revenue, this was on  just one occasion, to just one temple.’, Kitchen, ‘On the Reliability of the Old Testament’, pp. 133-134 (2003).

[20] ‘So a king of Assyria wrote to the Pharaoh about 1350 BC, ‘Gold is like dust in your land, one simply gathers it up.’ A contemporary king repeated this statement six times in letters to the Pharaoh! The Assyrian went on ‘Why do you think it is so valuable? I am building a new palace, send me enough gold to decorate it properly! When my ancestor wrote to Egypt, he was sent twenty talents of gold. . . . When (another king) wrote to Egypt to your father, he sent him twenty talents of gold . . . send me much gold!’ (Twenty talents by Babylonian standards was 600 kg or 11.7 cwts.).’, Millard, ‘Solomon In All His Glory’, Bible and Spade (11.2-3-4.73), 1982.

[21] ‘When Damascus surrendered to Adadnirari III, probably in 796 BC, the Assyrian received 2,300 talents of silver (69,000 kg; 67.76 tons), 20 talents of gold (600 kg; 1,320 lbs), and much else. Some sixty years later Tiglath-pileser III subjugated Samaria, placing Hoshea on the throne as his nominee. Samaria paid 10 talents of gold (300 kg; 660 lbs) as tribute (and an unknown amount of silver). The same emperor received the submission of Tyre, and with it the large sum of 150 talents of gold (4,500 kg; 4.4 tons).’, ibid., p. 74.

[22] ‘During the reign of Tuthmosis III the yield of the gold fields at Wawat in Nubia (the Sudan) for three years was 232.4 kg (512 lbs), 258.8 kg (570 lbs), and 286.1 kg (630 lbs). These may be exceptional figures, yet they show what sort of income was available from a single source. In the Annals of the same pharaoh, the booty taken between his twenty-second and his forty-second years amounted to over 11,500 kg (11.3 tons) of gold. His successor, Amenophis II (c. 1427–1401 BC) claimed the weight of gold vessels he took from the Levant was 6,800 deben (618.5 kg; 1,360 lbs).’, ibid., p. 75.

[23] ‘None of these figures approach the amounts recorded for Solomon except for the booty gathered by Tuthmosis III (11,500 kg; 11.3 tons).’, ibid., p. 75.

[24] ‘In Egypt Shishak’s successor Osorkon I gifted some 383 tons of gold and silver to the gods and temples of Egypt in the first four years of his reign, many of the detailed amounts being listed in a long inscription (now damaged) (figs. 22A, B). That sum would (in weight) be equivalent to almost seventeen years of Solomon’s annual gold revenue,’, Kitchen, ‘On the Reliability of the Old Testament’, p. 134 (2003).

[25] 1 Kings 1425 In King Rehoboam’s fifth year, King Shishak [Sheshonq] of Egypt attacked Jerusalem. 26 He took away the treasures of the LORD’s temple and of the royal palace; he took everything, including all the golden shields that Solomon had made.

[26]His reign is poorly documented, nothing hints at a far-reaching military adventure, bringing home rich booty.’, Millard, ‘Solomon In All His Glory’, Bible and Spade (11.2-3-4.76), 1982.

[27] ‘Osorkon’s father was Sheshonq 1 (c. 945–924 BC), the Shishak who took the gold from Solomon’s Temple in Jerusalem and from the Judaean treasury.’, ibid., p. 76.

[28] ‘Where could Osorkon have obtained such immense wealth, to spend on such a scale after only three and a third years of his reign? Barely five years earlier, Osorkon’s father Shishak had looted the wealth of Jerusalem. It seems unlikely to be a mere coincidence that almost immediately after that event Osorkon could dispose so freely of so much gold and silver.*The vast amounts of Solomon’s golden wealth may have ended up, at least in part, as Osorkon’s gift to the gods and goddesses of Egypt.’, Kitchen, ‘Where Did Solomon’s Gold Go?’, Biblical Archaeology Review (15.03), May/June 1989.

h1

What is archaeological ‘minimalism’?

April 26, 2011

Definition

The ‘minimalist’ view is that archaeology provides little or no support for the Biblical history.[1] [2] The best known adherents are Philip Davies,[3]  Lester Grabbe,[4] Niels Lemche,[5] Thomas Thompson,[6] and Keith Whitelam.[7] However, Israel Finkelstein and Neil Silberman are the only two prominent archaeologists associated with minimalist views.

Kenneth Kitchen

Kitchen[8] has raised numerous objections to minimalist claims, rejecting Thompson’s assertion that the Hebrew Tabernacle is a literary fiction,[9] that the Merneptah Stele is not reliable evidence for a people named ‘Israel’ in early 13th century Canaan,[10] that the Tel Dan Stele does not refer to a Hebrew ‘House of David’,[11] that the description of Solomon’s wealth is legendary,[12] and that the use of the first person perspective in the Mesha Stele indicates a post-mortem or legendary account.[13] [14] [15] Kitchen has also criticized Finkelstein and Silberman.[16] [17]

William Dever

Though far more sceptical than Kitchen, Dever[18] has nevertheless opposed minimalism vigorously. [19] [20] [21] [22]

Israel Finkelstein

Despite sympathies with some minimalist views, Finkelstein has rejected strongly the minimalist claims concerning Persian era Hebrew scribes,[23] that the ‘lists and details of royal administrative organization in the kingdom of Judah’ are fictional,[24] and that the Hebrew King David never existed.[25] [26] He has also acknowledged strong archaeological support for certain parts of the Biblical record.[27]

Amihai Mazar

With more in common with Finkelstein than the minimalists,[28] Mazar[29]  takes a moderate though critical view of the Biblical history.[30] [31]


[1] The ‘maximalist’ view is that archaeology overwhelmingly supports the Biblical history, and the moderate view is that archaeology substantially supports the Biblical history but that not all of the history can be supported directly from archaeology.

[2] ‘A recent trend in Syro-Palestinian archaeological study known as minimalism or revisionism suggests that Israel was created in the Hellenistic period.’, McCarty & Merrill, ‘Review: What Did the Biblical Writers Know and When Did They Know It? By William G. Dever.’, Bibliotheca Sacra (161.641.115), 2004.

[3] Biblical scholar.

[4] Historian.

[5] Biblical scholar.

[6] Biblical scholar.

[7] Biblical scholar.

[8] Egyptologist, Assyriologist, and archaeologist.

[9] ‘In so doing he ignores the whole of the comparative data that show clearly that the tabernacle was a product of Egyptian technology from the overall period 3000 to 1000 D.C. (plus Se-mitic analogues, 1900-1100), and would be unable to account for such facts.’, Kitchen, ‘On the Reliability of the Old Testament’, pp. 450-451 (2005).

[10] ‘The Israel of Merenptah’s stela was, by its perfectly dear determinative, a people (= tribal) grouping, not a territory or city-state; rare statements to the contrary are perverse nonsense, especially given the very high level of scribal accuracy shown by this particular monument.’, ibid., p. 451.

[11] ‘(i) The name “David” may be unusual, but is not unparalleled. Long centuries before, it was borne by a West Semitic chief carpenter in about 1730 B.C. on an Egyptian stela formerly in the collection at Rio de Janeiro. (ii) Dwd is neither the name (which Thompson admits) nor an epithet of a deity. Others are beloved of deities (for which references are legion!), but male deities are not beloved of others, human or divine (only goddesses are beloved of their divine husbands in Egypt). (iii) Mesha’s stela is ninth, not eighth, century. (iv) On Mesha’s stela dwd(h) is not a divine epithet of YHWH or anyone else. (v) Contrary to TLT, “House of X” does  mean a dynastic founder, all over the Near East, in the first half of the first millennium B.C.; it was an Aramean usage that passed into Assyrian nomenclature, and examples are common. (vi) Again, the expression, in part of its usage, is like the British “House of Windsor”, etc. Such usages were not peculiar to Aram, Assyria, and Judah either: in Egypt, the official title given to the Twelfth Dynasty (Turin Canon) was “Kings of the House (lit. ‘Residence’) of Ithet-Tawy” = ‘the Dynasty of Ithet-Tawy”. And the Thirteenth Dynasty was duly entitled “Kings who came after the [House of] King Sehetepibre” (founder of the Twelfth Dynasty). (vii) The charge of forgery is a baseless slur against the Dan expedition, without a particle of foundation in fact.’, ibid., pp. 452-453.

[12] ‘The point of the comparisons drawn with external (and firsthand!) sources was precisely that Solomon’s wealth (even as stated in Kings) was not exceptional or “fabulous/legendary” in its wider context. He was a pauper compared with (e.g.) Osorkon I, who, less than a decade after Solomon’s death, spent sums that massively outstrip Solomon’s stated income, and gave detailed accounts. The layering that TLT objects to was customary. At Karnak in Egypt, some temple columns were grooved to fit sheet gold from top to bottom, not mere “plastering.” As a touch of throwaway wealth, one need look no further than the recently discovered burials of two Assyrian queens. Solomon had just one golden throne? One pharaoh was sent ten at a time!’, ibid., p. 454.

[13] ‘Use of the first person by a monarch does not belong exclusively to either postmortem memorial texts or to later legends about such kings. A huge army of texts shows up the falsity of his presumption.’, ibid., p. 456.

[14]For first person, not postmortem, see (e.g.) Lipit-Ishtar (247), Warad-Sin/Kudur-mabuk (251-52), Rim-Sin I (253, his third text), Hammurabi (256-57), Ammi-ditana (258-59), and Shamshi-Adad I (259), all early second millennium. In the first millennium every major Assyrian king did exactly likewise, in various editions of their annals that were anything but postmortem, from Tiglath-pileser I to Assurbanipal (cf. ANET, 274-301; CoS II,261-306; RIMA, 1-3 passim).’, ibid., p. 456.

[15]Mesha’s stela is a contemporary building plus victory text, exactly like so many other inscriptions in the ancient Near East in all places and periods; the first-person formulation is irrelevant to its status, as the foregoing examples (far from exhaustive!) show.’, ibid., p. 457.

[16] ‘His reevaluation of the realm of Omri and Ahab is refreshing but wildly exaggerated, especially in archaeological terms. As others have shown amply, the redating will not work (cf. chap. 4, sec. 3 above).’, ibid., p. 464.

[17]The Philistines of Gerar (not those of the Pentapolis!) are a very different lot from the Iron Age group of that name. The term is a probably twelfth-century one substituted for Caphtorim or the like, precisely as Dan was substituted for Laish in Gen. 14:]4.’, ibid., p. 465.

[18] Archaeologist.

[19] ‘There are some who claim that the Bible contains little or no historical information about ancient Israel. I want to combat these “minimalist” or “revisionist” views of the history of ancient Israel by showing how archaeology can and does illuminate a historical Israel in the Iron Age of ancient Palestine (roughly 1200–600 B.C.E.)’, Dever, ‘Save Us from Postmodern Malarkey’, Biblical Archaeology Review (26.02), March/April 2000.

[20]Davies does not even cite the standard handbook, Mazar’s Archaeology of the Land of the Bible,’, Dever, ‘Who Were the Early Israelites? And Where Did they Come From?’, p. 138 (2003).

[21] ‘That same year Whitelam wrote an article for the Sheffield Journal for the Study of the Old Testament on “the realignment and transformation of Late-Bronze-Iron Age Palestine.” It was so full of caricatures of modern archaeological theory and results that I felt compelled to answer it in the same journal.’, ibid., p 139.

[22] ‘Thus he [Thompson] published two years later his revisionist treatment of ancient Israel: The Mythic Past: Biblical Archaeology and the Myth of Israel. Despite its subtitle, this work has next to nothing to do with real archaeology.’, ibid., p. 141.

[23] ‘First of all, as the biblical scholar William Schniedewind has indicated, literacy and extensive scribal activity in Jerusalem in the Persian and early-Hellenistic periods were much less influential than in the seventh century b.c.e. The assumption is inconceivable that in the fifth, or fourth, or even second centuries b.c.e., the scribes of a small, out-of-the-way temple town in the Judean mountains authored an extraordinarily long and detailed composition about the history, personalities, and events of an imaginary Iron Age “Israel” without using ancient sources.’, Finkelstein, ‘Digging for the Truth: Archaeology and the Bible’, in Schmidt (ed.), ‘The Quest for the Historical Israel’, Archaeology and Biblical Studies, number 17, p. 13 (2007).

[24] ‘The sheer number of name lists and details of royal administrative organization in the kingdom of Judah that are included in the Deuteronomistic History seems unnecessary for a purely mythic history. In any event, if they are all contrived or artificial, their coincidence with earlier realities is amazing.’, ibid., p. 13.

[25] ‘This argument suffered a major blow when the Tel Dan basalt stele was discovered in the mid-1990s.’, ibid., p. 14.

[26] ‘This was the first time that the name “David” was found in any contemporary source outside the Bible, in this case only about a century after his own supposed lifetime.’, ibid., p. 14.

[27] ‘Archaeological excavations and surveys have confirmed that many of the Bible’s geographical listings—for example, of the boundaries of the tribes and the districts of the kingdom—closely match settlement patterns and historical realities in the eighth and seventh centuries b.c.e. Equally important, the biblical scholar Baruch Halpern showed that a relatively large number of extra-biblical historical records—mainly Assyrian—verify ninth- to seventh-century b.c.e. events described in the Bible: the mention of Omri in the Mesha stele, those of Ahab and Jehu in the Shalmaneser III inscriptions, Hezekiah in the inscriptions of Sennacherib, Manasseh in the records of Esarhaddon and Ashurbanipal, and so on. No less significant is the fact, as indicated by the linguist, Avi Hurwitz, that much of the Deuteronomistic History is written in late-monarchic Hebrew, which is different from the Hebrew of post-exilic times.’, ibid., pp. 13-14.

[28]‘ Our views differ on certain important issues, but we share more in common than we do with either of the two extreme groups described above.’, Mazar, ‘On Archaeology, Biblical History, and Biblical Archaeology’, in Schmidt (ed.), ‘The Quest for the Historical Israel’, Archaeology and Biblical Studies, number 17, p. 29 (2007).

[29] Archaeologist.

[30] ‘My own choice is to follow those who claim that the initial writing of the Torah (the Pentateuch or Tetrateuch), of the Deuteronomistic History and large parts of the prophetic and wisdom literature took place during the late monarchy (eighth to early-sixth centuries b.c.e.), while during the exilic and post-exilic periods they underwent further stages of editing, expansion, and change. Yet, I also accept the

view of many scholars that the late-monarchic authors utilized earlier materials and sources.’, ibid., p. 29.

[31] ‘Both Assyrian inscriptions and local inscriptions like the stelae of Mesha, king of Moab, and of Hazael, king of Damascus (better known as the Tel Dan inscription), confirm that the general historical framework of the Deuteronomistic narrative relating to the ninth century was based on reliable knowledge of the historical outline of that century. Our understanding of the periods preceding the ninth century is of course foggier.’, ibid., p. 30.

h1

Were camels domesticated in the time of Abraham?

April 24, 2011

The Challenge

WF Albright, one of the most famous 20th century archaeologists, argued that the camel was not domesticated until around the 1st millennium, well after the time of Abraham.[1] This was considered persuasive by many Biblical scholars, who were convinced that references in Genesis to camels in Egypt during the time of Abraham[2] are anachronistic. [3] [4] [5]

The Evidence

Some evidence alleged for very early camel use in Mesopotamia has proved dubious,[6] [7] but Albright overlooked evidence for camel domestication reported by the French archaeologist Petrie in 1907.[8]

However, Petrie’s evidence for camel domestication during the Ramesside era in Egypt (1292-1069 BCE), was still too late for Abraham (from around 1900 BCE), though significantly earlier than Albright’s date.

Evidence for early camel domestication elsewhere in the Ancient Near East and North Africa is well documented, [9] [10] [11] [12] and has been used to argue defend the Genesis account. [13] [14] [15] [16]

It is recognized domesticated camel caravans must have passed through Egypt at an early date, even though the Egyptians made no reference to them at this time. [17] [18]  Bulliet observes that evidence for the early domestication of the camel in Mesopotamia cannot be ignored on the basis of their absence in Egyptian evidence.[19]

He agrees with Albright that evidence for Syrian domestic camel use during the 3rd to 2nd millennium is absent,[20] and argues the undisputed evidence of their use elsewhere in Mesoptamia indicates they entered the area on a very small scale as pack animals by rich traders, rather than being herded in large numbers.[21] [22]

Firm evidence for very early camel domestication in Egypt has caused some scholars to reconsider the Biblical narrative.[23] [24] [25]


[1]According to Albright, any mention of camels in the period of Abraham is a blatant anachronism, the product of later priestly tampering with the earlier texts in order to bring more in line with altered social conditions. The Semites of the time of Abraham, he maintains, herded sheep, goats, and donkeys but not camels, for the latter had not yet been domesticated and did not really enter the orbit of Biblical history until about 1100-1000 BC with the coming of the Midianites, the camel riding foes of Gideon.’, Bulliet, ‘The Camel and the Wheel’, p. 36 (1990 ed., originally published 1975).

[2] Genesis 12: 15 When Pharaoh’s officials saw her, they praised her to Pharaoh. So Abram’s wife was taken into the household of Pharaoh, 16 and he did treat Abram well on account of her. Abram received sheep and cattle, male donkeys, male servants, female servants, female donkeys, and camels.

[3] ‘Some scholars have suggested that only with the 1st millennium B.C. was the camel fully domesticated’, Pratico, ‘Nomadism’, in Bromiley (ed.), International Standard Bible Encyclopedia, volume 3, p. 547 (rev. ed. 1988).

[4]The almost unanimous opinion of Biblical scholars is that mention of domesticated camels in the Patriarchal narratives (Gn 12:16; 24:10; 30:43) constitutes an anachronism. Camels, they say, were not domesticated until late in the second millennium BC, centuries after the Patriarchs were supposed to have lived.’, Caesar, ‘Bible and Spade (13.77), 2000.

[5] ‘There continue to be some scholars who follow Albright’s skepticism (1942; 1945; 1949: 207) that references to camels in the patriarchal narratives are anachronistic (e.g. Koehler-Rollefson 1993: 183).’,  Younker, ‘Bronze Age Camel Petroglyphs In The Wadi Nasib, Sinai’, Bible and Spade (13.75), 2000.

[6] ‘To be sure, one or two representations of camels from early Mesopotamia have been alleged, but they are all either doubtfully camelline, as the horsy looking clay plaque from the third dynasty of ur (2345-2308 B.C.), or else not obviously domestic and hence possibly depictions of wild animals,’, Bulliet, ‘The Camel and the Wheel’, p. 46 (1990 ed., originally published 1975).

[7] ‘These five pieces of evidence, needless to say, may not convince everyone that the domestic camel was known in Egypt and the Middle East on an occasional basis between 2500 and 1400 B.C. Other early depictions, alleged to be of camels, which look to my eyes like dogs, donkeys, horses, dragons or even pelicans, might be more convincing to some than the examples described above.’, ibid., p. 64.

[8]The pottery figure of a camel laden with water-jars was found in a tomb of the XIXth dynasty in the northern cemetery. There were no traces of a later re-use of the tomb; the style of the figure is of the rough fingered pottery of the XIXth dynasty, and quite unlike any of the moulded Roman figures; and the water-jar, is of the XIVIIIth-XIXth dynasty type and not of a form used in Greek or Roman times. Hence it is impossible to assign this to the age when the camel is familiar in Egypt, and it shows that as early as Ramesside times it was sufficiently common to be used as a best of burden.’, Petrie, ‘Gizeh and Rifeh’, in ‘Publications of the Egyptian Research Account and British School of Archaeology in Egypt’ (13.23), 1907.

[9] ‘Camels are not anachronistic in the early second millennium BC, but find only sparing attestation and use both in Genesis and external sources then and until the twelfth century BC.’, Kitchen, ‘Historical Method and Hebrew Tradition’, Tyndale Bulletin (17.1.83), 1966.

[10] ‘Both the dromedary (the one-humped camel of Arabia) and the Bactrian camel (the two-humped camel of Central Asia) had been domesticated since before 2000 BC.’, Scarre, ‘Smithsonian Timelines of the Ancient World’, p. 176 (1993).

[11] ‘As far as hard dates go, the 2500-1500 B.C. suggested earlier for the introduction of the camel into Somalia is the best that can be done from available data. Given the stage domestication had reached by the time the camels and their owners crossed the sea, some additional time must be allowed for earlier stages. Taking this into consideration, it is easily conceivable that the domestication process first got underway between 3000 and 2500 B.C.’, Bulliet, ‘The Camel and the Wheel’, p. 56 (1990 ed., originally published 1975).

[12]Found in a context datable to 2700 B.C., the remains led the excavators to argue that camel domestication began in Turkmenia and spread south (Compagnoni and Tosi 1978: 95–99). The domestic camel was apparently known to the inhabitants of the Indus Valley Civilization by 2300 B.C., although the species utilized remains open to question (Meadow 1984: 134 and references).’,  Zarins, ‘Camel’, in Freedman (ed.), Anchor Yale Bible Dictionary (electronic ed. 1996).

[13]Archeological discoveries have now shown clearly that references to domesticated camels in Genesis are by no means anachronistic, as some earlier scholars supposed. While camel caravans seem to have been used regularly only from the Late Bronze Age onward, archeologists have found numerous bones of domesticated camels. Thus when Parrot was excavating Mari, he found camel bones in the ruins of a house dated in the pre-Sargonic period (ca 2400 B.C.). An eighteenth-century-B.C. relief from Byblos pictured a camel in a kneeling position, and a socket on the back showed that the animal’s hump and its load had been attached separately. In accord with patriarchal traditions, cylinder seals from Middle Bronze Age Mesopotamia showed riders seated upon camels.’, Harrison, ‘Genesis’, in Bromiley (ed.), International Standard Bible Encyclopedia, volume 3, p. 547 (rev. ed. 1988).

[14] ‘Excavations in eastern Arabia, an area once believed to be a cultural backwater unworthy of archaeological investigation, have turned up evidence that camels were first domesticated by Semites before the time of Abraham. Much of this evidence has been examined by M. C. A. MacDonald of the Oriental Faculty at the University of Oxford’, Caesar, ‘Bible and Spade (13.77), 2000.

[15] ‘the principle area of extensive early camel domestication was the Syro-Arabian desert, due west of Ur, Abraham’s birthplace (1995: 1356).’, ibid., p. 77.

[16] ‘possession of camels by Semitic travelers endowed them with a special advantage over those who did not, particularly in economic and political terms. This conforms to the Genesis image of the Patriarchs as wealthy, respected individuals who could hold their own against monarchs and chieftains.’, ibid., p. 78.

[17]Horses and camels were not represented in Old Kingdom Egypt and camels are said to have been introduced into Egypt much later than horses.’, Daly, ‘Egyptology: the missing millennium: ancient Egypt in medieval Arabic writings’, p. 102 (2005).

[18] ‘In view of the very early caravan links between Arabia and the Nile Valley, it would be very surprising if the camel had not reached Egypt before the first millennium BC; doubtless there were religious reasons for the lack of representations of this animal earlier than this. Camels could have been first introduced to Egypt from 1680 BC by the invading Hyksos, but it is not until the end of the second millennium that references to them begin to be found;’, Fage, ‘The Cambridge history of Africa: From the earliest times to c. 500 BC’, volume 1, pp. 288-289 (1982).

[19]Yet it is very difficult to explain away all of the evidence pointing to the camel’s presence outside the Arabian peninsula prior to the year 1400B.C. The effort is better spent looking into the reasons why the evidence from this early period is so very scarce.’, Bulliet, ‘The Camel and the Wheel’, p. 36 (1990 ed., originally published 1975).

[20] ‘The archaeological record, as Albright affirms, shows no indication of camel use in the Syrian area during the period in question, 2500-1400B.C.,’, ibid., p. 64.

[21] ‘Indeed, they must have played little or no part in the ordinary herding economy of the time. The most satisfactory explanation of this circumstance is that the camel was known because it was brought into the area by traders carrying goods from southern Arabia but that it was not bred or herded in the area. It is worthy of note that whereas the citations from the Bible associating camels with Abraham and his immediate descendants seem to fit the generalized pattern of later camel use in the area, they could also fit a pattern in which camels were very uncommon. The largest number of animals mentioned in those episodes is ten, and those ten are probably most of what Abraham had’, ibid., pp. 64-54.

[22]But it has been demonstrated that the camel was already in use during the period in question and that its probable homeland was southern Arabia. It is much more reasonable, therefore, to assume that the camel was the main carrier on the incense route from the very beginning, or nearly so, and that the Semitic tribes of the north came to know the camel in this way in very small numbers. In other words, the presence of camels in the Abraham story can be defended and the story treated as primary evidence of camel use without disputing Albright’s contention that camel-breeding nomads did not exist in Syria and northern Arabia at that time.’, ibid., pp. 66-67.

[23] ‘However, in various parts of the country some evidence for the presence of camels has been uncovered, associated with dates as far back as the predynastic period (Free 1944:191).’, Daly, ‘Egyptology: the missing millennium : ancient Egypt in medieval Arabic writings’, p. 102 (2005).

[24] ‘In the Egyptian Fayum province was found a camel-skull dated to the ‘Pottery A’ stage, i.e. within the period c. 2000–1400 BC, the period from the Patriarchs practically to Moses; see O. H. Little, Bulletin de l’Institut d’Égypte 18, 1935–6, p. 215.’, Kitchen, ‘Camel’, in Wood & Marshall (eds.), ‘New Bible Dictionary’, p. 160 (3rd ed. 1996).

[25] ‘However, there is now a growing body of scholars who believe that camel domestication must have occurred earlier than previously thought (prior to the 12th century BC) and that the patriarchal narratives accurately reflect this (e.g., Ripinsky 1984; Coote and Whitelam 1987: 102; Zarins 1992: 826; Borowski 1998: 112–18).’, Younker, ‘Bronze Age Camel Petroglyphs In The Wadi Nasib, Sinai’, Bible and Spade (13.75), 2000.

h1

Archaeology 2: Hezekiah’s Tunnel

February 20, 2011

Discovery of the tunnel built by King Hezekiah to provide water to Jerusalem in time of siege (2 Kings 22:20 and 2 Chronicles 32:30), was confirmed by an accompanying inscription which dates to the reign of Hezekiah.[1]

Minimalist scholars[2] John Rogerson and Philip Davies claimed that the inscription does not date to the reign of Hezekiah, but to the Hasmonean era (less than two centuries before the birth of Christ), a claim used to cast doubt on the date of the tunnel itself, and to argue that it was not built by Hezekiah.

Although acknowledging that their view is contradicted by the unanimous consent of palaeographers, [3] Rogerson and Davies claim that palaeography is insufficiently precise to differentiate between 8th century and 2nd century texts.[4] [5]

Ronald Hendel (a professional epigrapher specializing in Semitic languages), has responded strongly to the following claims made by Rogerson and Davies, demonstrating that they are in error. [6] [7] [8] [9]

Frank Cross, (Professor of Hebrew and Other Oriental Languages at Harvard University), observed that Rogerson and Davies were unqualified to make judgments on the text.[10] Professional epigraphist P. Kyle McCarter Jr made a similar statement.[11]

André Lemaire (specializing in First Temple period Old Hebrew inscriptions), objected that Rogerson and Davies appeal to outdated scholarship. [12] Esther Eshel (renowned epigraphist), rejected the claim that palaeography was too imprecise to date the inscription reliably. [13]

Professor of Biblical Hebrew and Northwest Semitic epigraphy at Harvard University Jo Hackett made the same argument. [14] Avi Hurvitz (professor of Bible and Hebrew linguistics), observed that the claims of Rogerson and Davies had been rejected by the leading epigraphists,[15] and disproved their linguistic arguments.[16] [17] [18]

Leading palaeographer Ada Yardeni dismissed the claim that the inscription shows evidence of a Hasmonean dating.[19]


[1] ‘Discovered by some boys at play in 1880, the Siloam Inscription commemorates the dramatic meeting of two teams of tunnelers, digging from opposite directions, during the construction of the tunnel in the reign of Hezekiah. The text, written in paleo-Hebrew, offers an unusual contrast to the Biblical account (2 Kings 22:20 and 2 Chronicles 32:30). Like most ancient commemorative texts, the Biblical account gives the royal perspective, whereas the Siloam Inscription features the style and content of a man who witnessed and participated’, Parker, ‘Jerusalem’s Underground Water Systems Siloam Inscription Memorializes Engineering Achievement’, Biblical Archaeology Review (20.04.), 2004.

[2] The ‘minimalist’ view is that archaeology provides little or no support for the Biblical history, the ‘maximalist’ view is that archaeology overwhelmingly supports the Biblical history, and the moderate view is that archaeology substantially supports the Biblical history but that not all of it can be supported directly from archaeology.

[3]They acknowledge that specialists in palaeography unanimously date the inscription to the last quarter of the eighth century BCE, but they maintain that the palaeographers are mistaken, apparently deluded by circular reasoning and professional hubris. This is a remarkable claim and deserves some consideration.’, Hendel, ‘The Date of the Siloam Inscription: A Rejoinder to Rogerson and Davies’, The Biblical Archaeologist (59.4.233), December 1996).

[4]Rogerson and Davies’ chief contention is that palaeographic analysis of ancient Hebrew inscriptions is extremely imprecise. In the case of the Siloam Inscription, they write: “the fact is this: it is frequently not possible to prove on paleographical evidence alone whether a text in paleo-Hebrew dates from, say, the eighth-seventh centuries or is Hasmonean or later”  (1996:146, italics in original).’, ibid., p. 233.

[5] ‘The inconclusiveness of the paleography is crucial to their larger argument that the Siloam Tunnel dates to the Hasmonean era.’, ibid., p. 233.

[6] ‘A review of the relevant evidence, however, shows that Rogerson and Davies’ paleographic arguments are deeply flawed. It is in fact quite easy to tell that the script of the Siloam Inscription belongs to the eighth-seventh century sequence and not to the paleo-Hebrew sequence of the Hasmonean era and later.’, ibid., p. 233.

[7] He rejects the clam that some of the letters in the text have no parallels in Iron Age inscriptions, casting doubt on the idea that they were written during the Iron Age; ‘The problem with this statement is that there are plenty of parallels to these four letters in Hebrew inscriptions from the late Iron Age, a number of which are datable by their archaeological context’, ibid., pp. 233-235.

[8] He rejects the claim that the script of the Siloam Inscription is closest to 4QpaleoExodm, one of the Dead Sea Scroll texts, dating to the first century; ‘In addition to the differences in zwaw, yod, kap, and qop, there are noticeable differences in dalet, lamed, mem, ‘ayin, and pe. Several other letters have more subtle differences in length, proportion, or stance. The reason for these differences in script is easy to ascertain: the letters in 4QpaleoExodm belong to a different (and later) stage in the historical development of Hebrew script than the letters in the Siloam Inscription. The paleo-Hebrew scripts of the Hasmonean era and later have undergone noticeable development in comparison to the scripts of the eighth-seventh century BCE.’, ibid., pp. 235-236, ‘I have gathered five instances of this sequence from inscriptions from the eighth-seventh century 3CE and one instance from 4QpaleoExodm (see chart on facing page). It is quite easy to see that the 4Q script is the odd one out and clearly differs from the eighth-seventh century BCE scripts. In contrast, the Siloam Inscription clearly belongs in the company of the other eighth-seventh century BCE inscriptions.’, ibid., p. 236.

[9] He rejects the claim that some linguistic features of the text are incongruous with an Iron Age date; ‘It is worth noting that Rogerson and Davies’ linguistic comments on Siloam inscriptions are also unwarranted. They state that “some of the linguistic features of the Siloam Inscription become problematic if it is early” (1996:146). These features are the apparent internal matres in lilwd and int’s.’ and the pronominal suffix of til, (where one would expect a final he). These forms are easily comprehended by the following observations. 1) lblwd and niiw’s may be consonantal spellings with the dipthong Iwl (so Cross and Freedman 1952:50-51), or they may be early examples of internal matres in the Siloam inscription, as found sporadically in other eighth century BCE inscriptions (Royal Steward, some Ihlk seals, etc.; see Sarfatti 1982:58-63).’, ibid., p. 236.

[10] ‘The list of significant features differentiating Old Hebrew from paleo-Hebrew can be extended to most, if not all, letters of the alphabet. To identify them requires an eye and memory for form, gifts that make the paleographer. Without such gifts, a scholar is in the same straits as the tone-deaf musician who wishes to conduct an orchestra.’, Cross, ‘Because They Can’t See a Difference, They Assert No One Can’, Biblical Archaeology Review (23.02.), 1997.

[11]No epigraphist trained in the scripts of these periods would confuse second-century B.C.E. paleo-Hebrew with sixth-century B.C.E. Hebrew, much less with eighth-century B.C.E. Hebrew.’, McCarter Jr, ‘No Trained Epigraphist Would Confuse the Two’, Biblical Archaeology Review (23.02.), 1997.

[12] ‘Because all Hebrew epigraphers now date the Siloam Inscription to the eighth century B.C.E., Rogerson and Davies are obliged to go back nearly a century for authority.† Of course, this earlier generation of scholars could not have been aware of the numerous Hebrew inscriptions from the First Temple period discovered since then.’, Lemaire, ‘Are We Prepared to Raze the Edifice?’, Biblical Archaeology Review (23.02.), 1997.

[13] ‘These examples, as well as many others, show that paleography stands on a strong and stable foundation. Today paleography can date documents to within half a century. It is true that paleography alone can only tell us that the Siloam Inscription may have been written at the end of the eighth century or in the seventh century B.C.E.,† but paleography can tell us with certainty that the inscription was not written in the second century B.C.E., as Rogerson and Davies “strongly suggest.”.’, Eshel, ‘Some Paleographic Success Stories’, Biblical Archaeology Review (23.02.), 1997.

[14] ‘Rogerson and Davies’s argument assumes that paleographers (neither Rogerson nor Davies is known as a paleographer) cannot tell the difference between pre-Exilic Old Hebrew and post-Exilic archaizing paleo-Hebrew. But they are wrong—very wrong. The science of paleography—the dating of scripts by the shape, form, stance, stroke order, and direction, as well as by other telltale diagnostic indications—can now date these scripts within a century and sometimes even closer. Contrary to Rogerson and Davies, paleographers can distinguish between pre-Exilic Old Hebrew and post-Exilic paleo-Hebrew. Rogerson and Davies admit, in fact, that the Siloam Inscription’s waw, yod, kap and qop do not fit well into a second-century B.C.E. script chart, and this should have been enough to tip them off to the problem with their argument.’, Hackett, ‘Spelling Differences and Letter Shapes Are Telltale Signs’, Biblical Archaeology Review (23.02.), 1997.

[15] ‘I am not surprised that some of the leading paleographical authorities in our field have so severely criticized the effort of Rogerson and Davies to place the Siloam Inscription in the Hasmonean period.’, Hurvitz, ‘Philology Recapitulates Paleography’, Biblical Archaeology Review (23.02.), 1997.

[16]The Hebrew of the Siloam Inscription is worlds apart from the Hebrew of the Dead Sea Scrolls or the Hebrew of the apocryphal book known as Ben Sira (also known as Ecclesiasticus or “The Wisdom of Jesus, Son of Sira”). To anyone versed in Hebrew linguistics, the Siloam Inscription clearly does fall under the heading of classical Hebrew, as manifested in classical Biblical literature.† It is true that the linguistic tools at our disposal cannot tell us whether the Siloam Inscription specifically reflects the time of Hezekiah’s rule (727–698 B.C.E.). On the basis of both the Biblical and post-Biblical evidence, however, we can conclude that—linguistically—the inscription must be dated to the classical phase of ancient Hebrew, that is, to the pre-Exilic period (before 586 B.C.E.).’, ibid.

[17] ‘Secondly, they argue that “the Chronicler’s Hebrew can mean that Hezekiah closed off the pool formed by the spring” (italics added). This is, indeed, a surprising suggestion. It cannot be admitted in a serious philological discussion. It is simply not what the Hebrew text says. To use Rogerson and Davies’s own wording, this suggestion is at best a “paraphrastic translation.” Worse, their suggestion violates a basic methodological ground-rule of any philological analysis: that the outcome of that analysis should not be inferred from—let alone dictated by—considerations lying outside the domain of philology.’, ibid.

[18] ‘In sum, it is the Biblical and inscriptional evidence adduced by Rogerson and Davies in support of their claim that undermines it. I would strongly suggest, therefore, that if they insist on their theory regarding the late dating of the Siloam tunnel, they should drop the linguistic argumentation from their discussion—which for them is unfamiliar territory.’, ibid.

[19] ‘If the Siloam Inscription were inscribed in the Hasmonean period, its script would reveal a late stage of evolution (like the paleo-Hebrew scrolls) or artificial archaized characteristics (like the Hasmonean coins). It displays neither.’, Yardeni, ‘They Would Change the Dates of Clearly Stratified Inscriptions—Impossible!’, Biblical Archaeology Review (23.02.), 1997.

h1

Archaeology 1: The Tell Dan Stele

February 19, 2011

The Tel Dan Stele is a piece of stone found in northern Israel [1] with an inscription referring to the ‘House of David’. It is a significant find, providing evidence that the nation of Israel existed as early as the 10th century, and that it was ruled over by king David, referred to in the Bible as the second king of Israel. [2] [3]

Several challenges have been made to the authenticity and translation of the Stele. [4] Philip Davies and Thomas Thompson are two scholars who have argued that the translation ‘House of David’ is incorrect. Professional archaeologists and epigraphers object to these reinterpretations, noting that they are suggested by Biblical scholars who have no formal qualifications in the relevant fields.

Philip Davies (an archaeological ‘minimalist’[5] who declares king David to be the literary invention of Jewish scribes in the Persian era), has claimed that the text, when properly translated, does not refer to the House of David. [6] [7] Since the Stele was not found in its original position (it was reused as building material in another location), [8] Davies has suggested it is actually a forgery. Secular archaeologist William Dever has completely dismissed this as impossible.[9]

Archaeologist and expert Assyriologist Kenneth Kitchen has exposed the errors of Thompson’s claims,[10] and the claims of Davies have been disproved by Anson Raineyd,[11] who commented ‘Davies and his “deconstructionists” can safely be ignored by everyone seriously interested in Biblical and Ancient Near Eastern studies’. [12]

Dever has noted that the majority of leading epigraphers believe the inscription to be a genuine reference to the House of David.[13] Even some of those who hold a minimalist view of the Bible and archaeology have conceded that the evidence for the authenticity of the inscription is overwhelming.

After reviewing a range of objections to the Stele’s authenticity, Niels Lemche concluded that the Stele should be accepted as genuine unless significant evidence to the contrary is found. [14] [15] Similarly, Lester Grabbe has acknowledged that the general scholarly consensus regards the Stele to be a genuine reference to the House of David. [16]


[1] The stele is broken; the first major piece was found in 1993, the second in 1994.

[2] ‘The new stela from Tel Dan was greeted with considerable enthusiasm, particularly as a tonic against denials that there had been an Israelite state in the tenth century B.C.E. Until the stela’s discovery, the formation of a state in Israel could not be dated later than the mid-ninth century B.C.E., because Assyrian epigraphs of the 850s and 840s B.C.E. and the roughly contemporary Mesha stela mentioned kings of Israel, some (Ahab, Omri, Jehu, and, later, Joash) by name.’, Halpern, ‘The Stela from Dan: Epigraphic and Historical Considerations’, Bulletin of the American Schools of Oriental Research (296.63), November 1994.

[3] ‘To William Dever and many other scholars, this inscription provides clear evidence that David was indeed a historical figure and not merely a mythical leader invented by later Biblical authors to give Israel a heroic past, as the Biblical minimalists maintain.’ Shanks, ‘Queries & Comments’, Biblical Archaeology Review (22.04), 2006.

[4] ‘Since the initial publication a small but vocal minority has objected to this interpretation, arguing that it was based on a “fundamentalist” reading of the inscription in light of the biblical text.’, Schniedewind, ‘Tel Dan Stela: New Light on Aramaic and Jehu’s Revolt’, Bulletin of the American Schools of Oriental Research (302.75), 1996.

[5] The ‘minimalist’ view is that archaeology provides little or no support for the Biblical history, the ‘maximalist’ view is that archaeology overwhelmingly supports the Biblical history, and the moderate view is that archaeology substantially supports the Biblical history but that not all of the history can be supported directly from archaeology.

[6] Davies, ‘‘House of David’ Built on Sand: The Sins of the Biblical Maximizers’ Biblical Archaeology Review (20.4), 1994.

[7] ‘Davies was faced with a decision—either he could admit that King David wasn’t invented by Persian-period scribes, or he could attempt to explain away the reference to “the House of David” as unrelated to the King David of the Bible. He chose the latter. This does not come as a surprise.’, Freedman & Geoghegan “House of David” Is There!’, Biblical Archaeology Review (21.2), 1995.

[8]‘The archaeological evidence is relatively straightforward: the fragment was incorporated in an inner gate structure that was destroyed in the mid-eighth century B.C.E. presumably in the 730s by Tiglath-Pileser III (Biran and Naveh 1993: 81-86). This means that the monument with which it originated was dismantled and broken up before the construction of the gate. Assuming that the inner gate stood for some time before its destruction, the construction of the gate structure would provide a stratigraphic date no later than the first half of the eighth century and no earlier than the midninth century. The excavator relates that “the level beneath” the fragment contained no pottery later than the mid-ninth century (Biran and Naveh 1993:

86). That is, the gate of the level in which the fragment was reused originates after a mid-ninthcentury “level.” The stela itself was earlier than the gate.’, Halpern, ‘The Stela from Dan: Epigraphic and Historical Considerations’, Bulletin of the American Schools of Oriental Research (296.68), 1994.

[9]I was there (Tel Dan) shortly after it was found. I’ve known Biran for 40 years. The woman who found it, Gila Cook, I hired at Hebrew Union College. I have handled the inscription. I know what I’m talking about. Theres no way (the Tel Dan Stele is inauthentic). All of this was covered by debris until he (A. Biran) started digging. True, it was found in secondary use. Nobody ever argued that it was in primary position. It was re-used in the wall. But there is no way in the world anybody could have dug down there, found that wall five years before Biran came along and planted it. Its impossible.’, Dever, in Shanks (ed.), ‘Face to Face: Biblical Minimalists Meet Their Challengers’, Biblical Archaeological Review (23.04), 1997.

[10] ‘(i) The name “David” may be unusual, but is not unparalleled. Long centuries before, it was borne by a West Semitic chief carpenter in about 1730 B.C. on an Egyptian stela formerly in the collection at Rio de Janeiro. (ii) Dwd is neither the name (which Thompson admits) nor an epithet of a deity. Others are beloved of deities (for which references are legion!), but male deities are not beloved of others, human or divine (only goddesses are beloved of their divine husbands in Egypt). (iii) Mesha’s stela is ninth, not eighth, century. (iv) On Mesha’s stela dwd(h) is not a divine epithet of YHWH or anyone else.’ ‘(v) Contrary to TLT, “House of X” does mean a dynastic founder, all over the Near East, in the first half of the first millennium B.C.; it was an Aramean usage that passed into Assyrian nomenclature, and examples are common. (vi) Again, the expression, in part of its usage, is like the British “House of Windsor”, etc. Such usages were not peculiar to Aram, Assyria, and Judah either: in Egypt, the official title given to the Twelfth Dynasty (Turin Canon) was “Kings of the House (lit. ‘Residence’) of Ithet-Tawy” = ‘the Dynasty of Ithet-Tawy”. And the Thirteenth Dynasty was duly entitled “Kings who came after the [House of] King Sehetepibre” (founder of the Twelfth Dynasty). (vii) The charge of forgery is a baseless slur against the Dan expedition, without a particle of foundation in fact.’, Kitchen, ‘On The Reliability Of The Old Testament’, pp. 452-453 (2003).

[11] Rainey (Professor Emeritus of Ancient Near Eastern Cultures and Semitic Linguistics at Tel Aviv University and expert in Semitic), ‘In response to Philip R. Davies’s brief article a few observations are in order. Davies represents what he and a circle of colleagues call the “deconstructionist” approach to Biblical traditions. The present instance can serve as a useful example of why Davies and his deconstructionists can safely be ignored by everyone seriously interested in Biblical and Ancient Near Eastern studies. Regarding the recently excavated Dan inscription, Davies makes a great quibble about the absence of the word divider between the components BYT (House) and DWD (David). Joseph Naveh and Avraham Birana did not explain the inscription in detail, perhaps because they took for granted that readers would know that a word divider between two components in such a construction is often omitted, especially if the combination is a well-established proper name. A well-known example of such a proper name composed of two components, is BL‘M.BRB‘R (Baalam, son of Beor) in line 4 of “Combination I” of the inscription from Deir ‘Alla. There is a word divider, a dot, between BL‘M (Balaam) and BRB‘R (son of Beor), but no word divider between BR (son [of]) and B‘R (Beor). The patronymic of the prophet Baalam consists of two vocables, BR (son [of]) and B‘R (Beor). These vocables are in the Semitic syntactical relationship known as “construct.” The first is closely attached to the second, which takes the accent for both. The House of David was certainly such a proper political and geographic name in the mid-ninth century B.C.E. André Lemaire’s recent discovery that the same name (BYTDWD) appears in the Mesha stela further confirms the reading in the Tel Dan inscription. The same situation pertains to BYTDWD (House of David) in the text from Dan. The first component is BYT (house), here in the “construct” form meaning “house of.” The main accent is on DWD (David), the second component. The combination was obviously recognized by the scribe of the Dan inscription as an important proper name. There is no reason whatever to doubt the correctness of the reading House of David.’, ‘The House of David and the House of the Reconstructionists’, Biblical Archaological Review (20.06), 1994.

[12] Ibid.

[13] ‘On the “positivist” side of the controversy, regarding the authenticity of the inscription, we now have published opinions by most of the world’s leading epigraphers (none of whom is a “biblicist” in Thompson’s sense): the inscription means exactly what it says.’ Dever, ‘What Did The Biblical Authors Know, And When Did They Know It?’, pp. 128-129 (2004).

[14] ‘Even if my observations about the almost uncanny prominence of the terms ‘King of Israel’ and ‘House of David’ are not accepted, I have to admit that the arguments in favour of seeing the Tel Dan fragments as fake need to be much more forceful—certainly stronger than I have been able to show in this survey—if they are to prove beyond doubt that the inscription is the work of a forger.’, Lemche, ‘House of David’: The Tel Dan Inscription(s)’, in Thompson & Jayyusi (eds.), ‘Jerusalem in Ancient History and Tradition’, p. 66 (2003).

[15] ‘At the end of the day, is the Tel Dan inscription important for the study of the history of Israel in Antiquity? Of course is important—if it is genuine. And, until the opposite has been proven, we have to reckon it to be genuine.’, ibid., p. 66.

[16] ‘The Tel Dan inscription generated a good deal of debate and a flurry of articles when it first appeared, but it is now widely regarded (a) as genuine and (b) as referring to the Davidic dynasty and the Aramaic kingdom of Damascus.’, Grabbe, ‘Reflections on the Discussion’, in Grabbe (ed.), ‘Ahab Agonistes: The Rise and Fall of the Omri Dynasty’, p. 333 (2007).